Personal Teleportation as a Weapon












15












$begingroup$


I've always considered teleportation to be one of the most under-rated super powers in all ways, but at the moment I'm thinking through the ramifications of personal teleportation as a superpower in fights both large and small. The "best" use I've seen for teleportation would possibly be Nightcrawler assaulting the White House in X-Men 2



In that case, teleportation is being used simply to give a large advantage in an otherwise normal fight. I suspect though that there are ways to use teleportation itself as a weapon, aka to injure/kill more directly than simply moving yourself around the battlefield to stab/punch/kick from unexpected locations. The only example I can think of where teleportation was used more directly as a weapon would be in Jumper when a Teleporter (aka someone who can teleport) teleported a moving bus directly at Samuel L. Jackson's character.



At the risk of becoming too broad, I'm interested in ways in which personal teleportation might be used as a weapon in "fights" both small and large, i.e.:




  1. Killing/incapacitating individuals

  2. Killing/incapacitating groups of men

  3. If possible, mass destruction on a large scale (blowing up cities?)


Obviously the options available to a Teleporter will depend on the mechanics of the teleportation. This is world building though, and I am indeed trying to build a world, so I'd like to leave the precise mechanics open. I would accept whatever mechanics allow my character to do the most damage. In general though, I'd limit it to two rules:




  1. The Teleporter can teleport themselves and/or any object (including people) they are touching, with a total mass limit of roughly double their own mass

  2. People/things/self can only be teleported to places which the Teleporter has previously visited in person


Given these limitations, just how much damage can my Teleporter do, and what will be an effective strategy to quickly end fights?










share|improve this question











$endgroup$








  • 2




    $begingroup$
    You can always end fights by fleeing. Though this might not be your desired answer, it is probably the best technique if you're not a martial arts master.
    $endgroup$
    – Erik
    16 hours ago






  • 3




    $begingroup$
    @Renan So if "absolute" velocity is maintained during teleportation, then you simply teleport someone to a different latitude or longitude and they instantly die... Sounds helpful! I guess that also means that the Teleporter is limited to short-distance "jumps" themselves - probably limited to teleporting within the horizon.
    $endgroup$
    – conman
    16 hours ago






  • 1




    $begingroup$
    @Renan Would that effect allow you to destroy cities? I'm thinking yes... Especially for the equator, teleporting large objects to the other side of the world would double the relative velocity (~1000 m/s). If you can teleport things as often as you would like (which is typically how the power works in movies/comics) then you can constantly teleport 200 pound blocks of concrete to the other side of the world until the area is effectively leveled...
    $endgroup$
    – conman
    16 hours ago






  • 3




    $begingroup$
    I am pretty sure there was an xmen movie where some bad dude with Nightcrawlery abilities was picking up government guys, teleporting with them, then leaving them to fall about 500 feet above the ground. Not super creative but effective.
    $endgroup$
    – Willk
    16 hours ago






  • 1




    $begingroup$
    @Willk I believe that was in Xmen: First Class
    $endgroup$
    – BKlassen
    15 hours ago
















15












$begingroup$


I've always considered teleportation to be one of the most under-rated super powers in all ways, but at the moment I'm thinking through the ramifications of personal teleportation as a superpower in fights both large and small. The "best" use I've seen for teleportation would possibly be Nightcrawler assaulting the White House in X-Men 2



In that case, teleportation is being used simply to give a large advantage in an otherwise normal fight. I suspect though that there are ways to use teleportation itself as a weapon, aka to injure/kill more directly than simply moving yourself around the battlefield to stab/punch/kick from unexpected locations. The only example I can think of where teleportation was used more directly as a weapon would be in Jumper when a Teleporter (aka someone who can teleport) teleported a moving bus directly at Samuel L. Jackson's character.



At the risk of becoming too broad, I'm interested in ways in which personal teleportation might be used as a weapon in "fights" both small and large, i.e.:




  1. Killing/incapacitating individuals

  2. Killing/incapacitating groups of men

  3. If possible, mass destruction on a large scale (blowing up cities?)


Obviously the options available to a Teleporter will depend on the mechanics of the teleportation. This is world building though, and I am indeed trying to build a world, so I'd like to leave the precise mechanics open. I would accept whatever mechanics allow my character to do the most damage. In general though, I'd limit it to two rules:




  1. The Teleporter can teleport themselves and/or any object (including people) they are touching, with a total mass limit of roughly double their own mass

  2. People/things/self can only be teleported to places which the Teleporter has previously visited in person


Given these limitations, just how much damage can my Teleporter do, and what will be an effective strategy to quickly end fights?










share|improve this question











$endgroup$








  • 2




    $begingroup$
    You can always end fights by fleeing. Though this might not be your desired answer, it is probably the best technique if you're not a martial arts master.
    $endgroup$
    – Erik
    16 hours ago






  • 3




    $begingroup$
    @Renan So if "absolute" velocity is maintained during teleportation, then you simply teleport someone to a different latitude or longitude and they instantly die... Sounds helpful! I guess that also means that the Teleporter is limited to short-distance "jumps" themselves - probably limited to teleporting within the horizon.
    $endgroup$
    – conman
    16 hours ago






  • 1




    $begingroup$
    @Renan Would that effect allow you to destroy cities? I'm thinking yes... Especially for the equator, teleporting large objects to the other side of the world would double the relative velocity (~1000 m/s). If you can teleport things as often as you would like (which is typically how the power works in movies/comics) then you can constantly teleport 200 pound blocks of concrete to the other side of the world until the area is effectively leveled...
    $endgroup$
    – conman
    16 hours ago






  • 3




    $begingroup$
    I am pretty sure there was an xmen movie where some bad dude with Nightcrawlery abilities was picking up government guys, teleporting with them, then leaving them to fall about 500 feet above the ground. Not super creative but effective.
    $endgroup$
    – Willk
    16 hours ago






  • 1




    $begingroup$
    @Willk I believe that was in Xmen: First Class
    $endgroup$
    – BKlassen
    15 hours ago














15












15








15


4



$begingroup$


I've always considered teleportation to be one of the most under-rated super powers in all ways, but at the moment I'm thinking through the ramifications of personal teleportation as a superpower in fights both large and small. The "best" use I've seen for teleportation would possibly be Nightcrawler assaulting the White House in X-Men 2



In that case, teleportation is being used simply to give a large advantage in an otherwise normal fight. I suspect though that there are ways to use teleportation itself as a weapon, aka to injure/kill more directly than simply moving yourself around the battlefield to stab/punch/kick from unexpected locations. The only example I can think of where teleportation was used more directly as a weapon would be in Jumper when a Teleporter (aka someone who can teleport) teleported a moving bus directly at Samuel L. Jackson's character.



At the risk of becoming too broad, I'm interested in ways in which personal teleportation might be used as a weapon in "fights" both small and large, i.e.:




  1. Killing/incapacitating individuals

  2. Killing/incapacitating groups of men

  3. If possible, mass destruction on a large scale (blowing up cities?)


Obviously the options available to a Teleporter will depend on the mechanics of the teleportation. This is world building though, and I am indeed trying to build a world, so I'd like to leave the precise mechanics open. I would accept whatever mechanics allow my character to do the most damage. In general though, I'd limit it to two rules:




  1. The Teleporter can teleport themselves and/or any object (including people) they are touching, with a total mass limit of roughly double their own mass

  2. People/things/self can only be teleported to places which the Teleporter has previously visited in person


Given these limitations, just how much damage can my Teleporter do, and what will be an effective strategy to quickly end fights?










share|improve this question











$endgroup$




I've always considered teleportation to be one of the most under-rated super powers in all ways, but at the moment I'm thinking through the ramifications of personal teleportation as a superpower in fights both large and small. The "best" use I've seen for teleportation would possibly be Nightcrawler assaulting the White House in X-Men 2



In that case, teleportation is being used simply to give a large advantage in an otherwise normal fight. I suspect though that there are ways to use teleportation itself as a weapon, aka to injure/kill more directly than simply moving yourself around the battlefield to stab/punch/kick from unexpected locations. The only example I can think of where teleportation was used more directly as a weapon would be in Jumper when a Teleporter (aka someone who can teleport) teleported a moving bus directly at Samuel L. Jackson's character.



At the risk of becoming too broad, I'm interested in ways in which personal teleportation might be used as a weapon in "fights" both small and large, i.e.:




  1. Killing/incapacitating individuals

  2. Killing/incapacitating groups of men

  3. If possible, mass destruction on a large scale (blowing up cities?)


Obviously the options available to a Teleporter will depend on the mechanics of the teleportation. This is world building though, and I am indeed trying to build a world, so I'd like to leave the precise mechanics open. I would accept whatever mechanics allow my character to do the most damage. In general though, I'd limit it to two rules:




  1. The Teleporter can teleport themselves and/or any object (including people) they are touching, with a total mass limit of roughly double their own mass

  2. People/things/self can only be teleported to places which the Teleporter has previously visited in person


Given these limitations, just how much damage can my Teleporter do, and what will be an effective strategy to quickly end fights?







combat teleportation






share|improve this question















share|improve this question













share|improve this question




share|improve this question








edited 16 hours ago







conman

















asked 16 hours ago









conmanconman

710215




710215








  • 2




    $begingroup$
    You can always end fights by fleeing. Though this might not be your desired answer, it is probably the best technique if you're not a martial arts master.
    $endgroup$
    – Erik
    16 hours ago






  • 3




    $begingroup$
    @Renan So if "absolute" velocity is maintained during teleportation, then you simply teleport someone to a different latitude or longitude and they instantly die... Sounds helpful! I guess that also means that the Teleporter is limited to short-distance "jumps" themselves - probably limited to teleporting within the horizon.
    $endgroup$
    – conman
    16 hours ago






  • 1




    $begingroup$
    @Renan Would that effect allow you to destroy cities? I'm thinking yes... Especially for the equator, teleporting large objects to the other side of the world would double the relative velocity (~1000 m/s). If you can teleport things as often as you would like (which is typically how the power works in movies/comics) then you can constantly teleport 200 pound blocks of concrete to the other side of the world until the area is effectively leveled...
    $endgroup$
    – conman
    16 hours ago






  • 3




    $begingroup$
    I am pretty sure there was an xmen movie where some bad dude with Nightcrawlery abilities was picking up government guys, teleporting with them, then leaving them to fall about 500 feet above the ground. Not super creative but effective.
    $endgroup$
    – Willk
    16 hours ago






  • 1




    $begingroup$
    @Willk I believe that was in Xmen: First Class
    $endgroup$
    – BKlassen
    15 hours ago














  • 2




    $begingroup$
    You can always end fights by fleeing. Though this might not be your desired answer, it is probably the best technique if you're not a martial arts master.
    $endgroup$
    – Erik
    16 hours ago






  • 3




    $begingroup$
    @Renan So if "absolute" velocity is maintained during teleportation, then you simply teleport someone to a different latitude or longitude and they instantly die... Sounds helpful! I guess that also means that the Teleporter is limited to short-distance "jumps" themselves - probably limited to teleporting within the horizon.
    $endgroup$
    – conman
    16 hours ago






  • 1




    $begingroup$
    @Renan Would that effect allow you to destroy cities? I'm thinking yes... Especially for the equator, teleporting large objects to the other side of the world would double the relative velocity (~1000 m/s). If you can teleport things as often as you would like (which is typically how the power works in movies/comics) then you can constantly teleport 200 pound blocks of concrete to the other side of the world until the area is effectively leveled...
    $endgroup$
    – conman
    16 hours ago






  • 3




    $begingroup$
    I am pretty sure there was an xmen movie where some bad dude with Nightcrawlery abilities was picking up government guys, teleporting with them, then leaving them to fall about 500 feet above the ground. Not super creative but effective.
    $endgroup$
    – Willk
    16 hours ago






  • 1




    $begingroup$
    @Willk I believe that was in Xmen: First Class
    $endgroup$
    – BKlassen
    15 hours ago








2




2




$begingroup$
You can always end fights by fleeing. Though this might not be your desired answer, it is probably the best technique if you're not a martial arts master.
$endgroup$
– Erik
16 hours ago




$begingroup$
You can always end fights by fleeing. Though this might not be your desired answer, it is probably the best technique if you're not a martial arts master.
$endgroup$
– Erik
16 hours ago




3




3




$begingroup$
@Renan So if "absolute" velocity is maintained during teleportation, then you simply teleport someone to a different latitude or longitude and they instantly die... Sounds helpful! I guess that also means that the Teleporter is limited to short-distance "jumps" themselves - probably limited to teleporting within the horizon.
$endgroup$
– conman
16 hours ago




$begingroup$
@Renan So if "absolute" velocity is maintained during teleportation, then you simply teleport someone to a different latitude or longitude and they instantly die... Sounds helpful! I guess that also means that the Teleporter is limited to short-distance "jumps" themselves - probably limited to teleporting within the horizon.
$endgroup$
– conman
16 hours ago




1




1




$begingroup$
@Renan Would that effect allow you to destroy cities? I'm thinking yes... Especially for the equator, teleporting large objects to the other side of the world would double the relative velocity (~1000 m/s). If you can teleport things as often as you would like (which is typically how the power works in movies/comics) then you can constantly teleport 200 pound blocks of concrete to the other side of the world until the area is effectively leveled...
$endgroup$
– conman
16 hours ago




$begingroup$
@Renan Would that effect allow you to destroy cities? I'm thinking yes... Especially for the equator, teleporting large objects to the other side of the world would double the relative velocity (~1000 m/s). If you can teleport things as often as you would like (which is typically how the power works in movies/comics) then you can constantly teleport 200 pound blocks of concrete to the other side of the world until the area is effectively leveled...
$endgroup$
– conman
16 hours ago




3




3




$begingroup$
I am pretty sure there was an xmen movie where some bad dude with Nightcrawlery abilities was picking up government guys, teleporting with them, then leaving them to fall about 500 feet above the ground. Not super creative but effective.
$endgroup$
– Willk
16 hours ago




$begingroup$
I am pretty sure there was an xmen movie where some bad dude with Nightcrawlery abilities was picking up government guys, teleporting with them, then leaving them to fall about 500 feet above the ground. Not super creative but effective.
$endgroup$
– Willk
16 hours ago




1




1




$begingroup$
@Willk I believe that was in Xmen: First Class
$endgroup$
– BKlassen
15 hours ago




$begingroup$
@Willk I believe that was in Xmen: First Class
$endgroup$
– BKlassen
15 hours ago










11 Answers
11






active

oldest

votes


















14












$begingroup$

The following assumes that, like in most universes, you cannot simply teleport part of an object or person. That would be... too easy.



Another assumption is that the power is "designed" to be safe for the user, so momentum, pressure etc. are adjusted reasonably well to the target frame of reference.



The environment is your greatest weapon



There aren't many threats that cannot be dealth with by teleporting them into mid-air, the deep sea, space, a sealed mine or a prison cell. Anything your character touches can be considered out of the fight. Sure, some of these locations are more exotic than others, but even if you want to keep the ability secret and cannot accept help from others, a sufficiently effective death trap can be improvised on a budget by e.g. renting a boat, buying a plane ticket or just crossing over a deep pit with a length of rope (that you remove afterwards).



Conventional weapons and transportation work just as well



Logistical limitations don't apply to you. Screw getting your own hands dirty, just provide an army of your choice with a steady stream of personnel, supplies and intel. That'll do more than a single person ever could.



If you don't like other people risking their necks for your cause, get a fast vehicle (refueling at will means unlimited range) or at least heavy body armor for "lucky bullet" protection and stock up on heavy weapons. You don't need to reload, you can switch between weapons and angles of attack pretty much at will and no one can ever take any territory from you because the second you feel like it, they'll sit on a pile of armed bombs. Still too risky? Make contact once, then teleport home and port in a steady stream of drones.



"Suicide" bombings are also an option, but I don't like those as much because you still need to get in range, and as we've already established, anything within touch range is doomed anyway. And anything a person could reach, a teleported missile or drone will have a much easier time with.



Never underestimate gravity



Assuming you're




  • ok at math

  • able to retain momentum when teleporting

  • able to afford a handful of trips to orbit


nowhere is really safe from orbital bombardment. You'll have to experiment a bit to work out what types of ammunition work best but if you want to flatten a city, that's probably the way to go.






share|improve this answer











$endgroup$













  • $begingroup$
    If you can teleport to where you've been with the momentum you had last time you were there, getting to orbit is incrementally is reasonably cheap.
    $endgroup$
    – Joshua
    12 hours ago










  • $begingroup$
    "That would be... to easy." Indeed.
    $endgroup$
    – reirab
    11 hours ago












  • $begingroup$
    To add to your list of environmental factors, teleporting someone into the environment - i.e. half way into a wall or directly into the middle of a block of concrete would be pretty effective as well. Or just teleport half of someone to somewhere else.
    $endgroup$
    – Aaron Lavers
    5 hours ago





















7












$begingroup$

So like Friendlysociopath used, I will be borrowing from anime to answer.



A Certain Scientific Teleporter



In the series A Certain Magical Index and A Certain Scientific Railgun we're introduced to a character who can teleport. They're a part of a neighborhood watch-like program (so they don't go about killing people) but they have an unusually high combat incident count. Their main line of attack is to teleport metal darts into the clothes of opponents and pin them to surfaces. Usually they knock them against the surfaces first (satisfying a more narrow application of the "Must have visited this place before" clause). But they have a brevity of application to their powers.




  1. They can teleport objects into people. So if you're a teleporter looking to just end any 1v1 engagement, teleport a needle into their heart or brain. Done. Assuming the "visited this place before" means only that you've physically seen it. But if we're more specific (e.g. you've had to have touched the place before") then we still have the standard option of restraining people with heavy objects once you've touched them.


  2. They can teleport material into structural supports and bring down buildings. Again, assuming you just have to physically have seen your target place before then you can just teleport glass panes from a building's windows to their structural supports and down goes a couple hundred million dollars not to mention the mass panic. Although, this involves changing the orientation of an object in teleportation. Another limitation. But if you have to touch all of the target you're teleporting to then bringing down buildings is a little out of reach. However, you can still cause mass panic because...



  3. You can teleport double your mass to the tops of buildings and have them come crashing down. 140kg dropping 20 stories will hurt a great deal. Again, if you only have to have seen the target, dropping 140kg asphalt balls from the cloud line will be incredibly damaging depending on cooldown times or other physical limitations. The character in question doesn't do this but a Rival teleporter does try to do this to crush the character.







So you have options in combat when it comes to being a teleporter. As others have said you can really abuse the rule and win a lot of fights just by sending an opponent free solo skydiving or jamming their own weapon into their brain. On a less lethal note, dropping 140kg's of sand on someone will buy you time to incapacitate them or kill them. Teleporting 140kg's of industrial staples will pin someone to a surface or fewer metal stakes will do the same. Even just threatening teleporting a needle into someone's brain will stop a lot of fights.



Against groups, like IT Alex stated, teleport them away till you get down to a manageable number then start incapacitating or killing as you see fit. If you have to fight full groups, hope for the environment to be in your favor and teleport holes into the ground in front of people and use that dirt to cover someone. Or start throwing javelins at people.



And if you're just in it for the mass panic, you have too many options to count. You can't blow up cities but you can royally mess with them. Teleport small pieces of bridge supports away, drop asphalt boulders from on high, ruin electrical substations, even destroy buildings. You can cause quite a bit of damage in a very short amount of time with little effort.






share|improve this answer









$endgroup$













  • $begingroup$
    Since you are assuming they don't need to see where they are teleporting, I'd also add teleporting something into the ground below a structure or into the infrastructure, such as natural gas or electrical lines. Anything sufficiently large will cause a massive explosion of earth, besides the disruption of the services.
    $endgroup$
    – computercarguy
    10 hours ago










  • $begingroup$
    The crux really lies on how you apply "I've visited this place in person before". This is why I gave two distinct ways to deal with problems. If we speak conservatively and you have to have touched the object and the target is a lot different than just having to have seen the target in the past. In this case, if we're being conservative about mass destruction, you can't really teleport something into the ground far enough for it to hit infrastructure (maybe artillery drop it). But liberally you can. It all depends on the rules.
    $endgroup$
    – GuidingOlive
    10 hours ago












  • $begingroup$
    Even if we aren't using mass destruction, simply a large mass appearing inside another mass will cause all kinds of stresses that will react similar to an explosion. The harder the material the better, in this case, since a sandy ground might just deform some, but even sufficiently hard limestone should rupture quite significantly. And yes, this would be liberal with where something can be transported, such as the center of a solid object.
    $endgroup$
    – computercarguy
    10 hours ago



















6












$begingroup$


The Teleporter can teleport themselves and/or any object (including people) they are touching, with a total mass limit of roughly double their own mass
People/things/self can only be teleported to places which the Teleporter has previously visited in person




If you've ever watched Darker Than Black the protagonist faces someone with a teleportation power. Except instead of teleporting themselves they teleport a fist-sized bunch of matter- which they typically use to swap pieces of building with the heart of the person they're trying to assassinate.






share|improve this answer









$endgroup$













  • $begingroup$
    "People/things/self can only be teleported to places which the Teleporter has previously visited in person" -- I'm pretty sure the Teleporter has not physically been inside their intended victim before, so swapping out internal organs (or even swapping things in) isn't going to happen I don't think.
    $endgroup$
    – Doktor J
    10 hours ago



















5












$begingroup$

Air Drop



In X-men: First Class, a guy with Nightcrawler's powers kills people by teleporting into the air with them and then dropping them while he teleports back down. Since he can teleport right up to someone to grab them, there isn't much defense against this. You specify only places he has been, but as long as he's gone skydiving even once, this shouldn't be a problem.



(Others mentioned that specific attack in the comments section. I just noticed, even thought they posted before I did. Credit should go to @Willk and @BKLassen. Or, should I just delete that part?)



Partial Teleport



If your teleporter doesn't need to take the whole object or person with him, he could put his hand on a guy, and teleport away with only part of them. Instant decapitations, or 'Disarming'.






share|improve this answer











$endgroup$













  • $begingroup$
    I'm just considering the implications of a Nightcrawler-like character who has gone skydiving exactly once and continues to use that strategy every time he's in a fight... The number of bodies found piled up in that one spot would start to get a little suspicious...
    $endgroup$
    – Darrel Hoffman
    13 hours ago






  • 1




    $begingroup$
    All the person has to do is take a flight across the Atlantic or Pacific. Heck, even a cruise ship will do but it's not as certain. Then they can teleport themselves and the target somewhere where no one will find the body. With the airline flight, the teleporter would be so high that he would have to deal with very low airpressure and extreme cold for a fraction of a second. Off of a cruise ship, the kill isn't certain since they arrive closer to the water and in a shipping lane.
    $endgroup$
    – ShadoCat
    13 hours ago






  • 2




    $begingroup$
    The Earth is rotating and orbiting the Sun, and the Milky Way (with the Earth and the Sun in it) is also rotating. Given the density of space, the location he has been to is now with very high probability in outer space, and humans can't survive the cold and the lack of air for more than a minute. This is practical for taking someone to outer space, and thus quickly killing them, but then teleporting back to a safe place will be a problem.
    $endgroup$
    – pts
    13 hours ago








  • 1




    $begingroup$
    @pts Currently we have no absolute reference frame, that is, no point in space which is absolutely "not moving". (We may find one in the future, but probably not.) So we only have relative reference frames to choose from. At the very least, Teleporters will teleport with reference to the Earth, if not with reference to their destination e.g. a moving vehicle.
    $endgroup$
    – Martin Carney
    12 hours ago



















4












$begingroup$

Unfortunately, the limitation of places he can teleport to by having to have been there at some point makes him a better defensive fighter than an offensive. Having sufficient time to prepare, he can literally attack from anywhere within an area.



One on One.



Your teleporter should teleport twice his mass in an entangling object over a wide area (nets, barbed wire, fishing line) to immobilize his opponent. Then, teleport a pointy weighted object on top of them. Having been entangled they will have a hard time dodging and each dodge they succeed will make them more entangled and less able to dodge the next one. Alternatively, teleport behind them and touch them to teleport them to a location that you have sky-dived (without a parachute).



Multiple opponents.



Survival should be paramount in a one on many fight. Teleport and touch a combatant and put them literally anywhere other than the fighting location until it is 1v1. You could even use the mid-air approach from previously. The key is to not fight disadvantageous fights and instead make every fight fair or in your favor.



Mass Destruction.



No more so than the average person. The only caveat is that what would normally be a suicide attack for most people is now survivable via teleportation.



Most people would be unable to track and/or stop your teleporter. If he can teleport as fast as he can think then he would be able to (with practice) eliminate people within seconds.






share|improve this answer









$endgroup$













  • $begingroup$
    Kamikaze teleporters sounds cool.
    $endgroup$
    – Malkev
    15 hours ago










  • $begingroup$
    Huh, You made me think of something. What if he concentrates hard enough to constantly teleport anything that touches him for the most miniscule of moments, therefore gaining invulnerability from shrapnel and bullets alike because they will not have enough time to impart energy!
    $endgroup$
    – IT Alex
    11 hours ago



















4












$begingroup$

Well, one less lethal thing no one else has pointed out is instant incarceration. The teleporter visits a jail cell or a bank vault, which is then locked. The teleporter doesn't even need to be let out, of course, because they can take themselves out at any time.



Once in conflict with someone, the teleporter need only touch their opponent to teleport them away, into the locked cell or vault. (Go to jail. Go directly to jail. Do not pass Go. Do not collect $200.)



For a more lethal version, the teleporter need only enter an incinerator when it's not on, and then have it fired up after they leave. (Tap. You're toast.) This idea could be extended to any dangerous environment where the danger can be easily turned off and on: inside an nuclear reactor, directly above an industrial metal shredder, a large container filled with acid or toxic gas, etc. Heck, the teleporter could even go in a deep sea submarine once, and then teleport people to the bottom of the ocean.






share|improve this answer









$endgroup$













  • $begingroup$
    The pragmatist in me would book a trip on the deepest submersible possible. It would guarantee death (I think the sudden pressure change would kill someone long before drowning) and dispose of the body simultaneously. However, I would rather book a trip into space so I can drop them into LEO. I expect they would fall back to Earth and leave a mess for people to investigate, but it would be way cooler! Besides, who doesn't want to go to space?
    $endgroup$
    – conman
    6 hours ago



















3












$begingroup$

My answer works on Three assumptions:



1: The person using the teleportation power can teleport parts of a object he/she is touching.



And/Or



2: There is little/no cool-down in-between teleportation attempts.



And/Or



3: If the teleporter can choose how precise the "location he has been" part is.



Most organisms cannot function in combat without a brain. So if assumption #1 is correct, then you can not only quickly kill them, but also merely incapacitate them by removing things such as clothing, weapons, limbs, or possibly just all the air in their lungs.



All people need a defense, whether it is armor, wits, luck, or a combination of said three. Teleporters have a fourth if assumption #2 works: By constantly teleporting the area near them away, you create what essentially is a wormhole that they control in order to not only prevent damage, but maybe also send the damage right back at the aggressors!



If you have in combat and holding a sword (example item), you could teleport the hilt of the sword right next to the opponent's forehead, with the blade orientated towards the brain. Voila! Sword has now been stabbed though whatever armor the guy has, and is now in the brain.



This answer focuses on on-on-one and can be extended to one-on-multiple.






share|improve this answer









$endgroup$





















    3












    $begingroup$

    This power would be extraordinarily useful. Anywhere this person has ever been, they can make "anything they touch" appear there.



    Convenient death trap: arrange to send folks to kill zones



    Make an expedition to the North Pole (which will kill probably even many supers unprepared to be sent there in very short order). Take an airplane ride with a window seat, then simply teleport anyone the person wants to get rid of there.



    Or just get a secret lair. Build a sub-sub basement. Surround it with chain link fencing, then flood it with water. Teleport people there to drown. Much less public that way. People just disappear and aren't found in spectacular, publicity-generating ways (like falling out of the sky randomly over the Midwest, or a huge pile of people killed via exposure being found at the North Pole).



    If the user doesn't want to kill their enemies, and has resources, like they're Batman, build a prison. Hire a bunch of superpowered guards. Spend a few minutes locked inside a 2-foot by 2-foot square closet surrounded by powerproof glass, and you've got an intake cell where your new inmate materializes. Repeat, and you can handle multiple incoming bad (or I suppose good) guys simultaneously.



    Anywhere the user has, or might have ever, been is not secure



    Anywhere they can break into, or might have ever broken into, is not secure. Any place they might have flim-flammed their way past the front desk in disguise (think civvies), even briefly, is not secure. Any place they've ever worked cannot be considered secure. Note that the list of such places is vast because you won't know where they've been. This remains true as long as they live.



    They can take a tour of the White House, even the unsecured parts, in disguise. Twenty years later, they could decide 'screw this, time to assassinate the President' and now you've got a problem. They can take a job as the janitor of an office building where someone they're targeting works. Failing everything else, they take a job working for these guys and then pretty much learn how to get anywhere they want.



    Seriously, watch the video, it exposes how much even 'secure' locks mostly aren't. All this person needs to do is pop a door when nobody's around, or maybe when people are around and you just do the 'elevator repairman' schtick (again from the video).



    TL;DR - This person can get almost anywhere. And if they can, then anyone who can engage their services can also. If you want in, and either can pay, or convince them to help you, a plan can almost certainly be arranged to give the teleport user the ability to get your entire crew in. This person can also assassinate a lot of folks by finding their office, then teleporting hand grenades or nerve gas canisters or even assassin droids in.






    share|improve this answer









    $endgroup$













    • $begingroup$
      Have their 'secret identity' - or one of their secret identities - explicitly be as a pen tester. Then you can essentially hand wave their ability to get somewhere that other folks would really rather they didn't.
      $endgroup$
      – Dayton Williams
      5 hours ago



















    2












    $begingroup$

    Other answers have covered fairly well the options for one to one combat, and even vs groups. This answer poses a solution to mass destruction.



    The only restriction is enabling partial teleportation.



    Here's a few ideas, depending on how extreme you want to go.



    1) Shockwaves. Teleportation should always produce a shockwave. Consider, what happens to the air that used to occupy the space the teleported object now does? As you've defined the ability (must be touching thing to teleport) they could not possibly swap the air and the teleported object. So the air must remain, pushed out at hyper velocity by the sudden presence of something teleporting in. Boom, shockwave!



    2) Since you are already moving matter out of the way (air) what happens if you teleport an object into something more... substantial? Imagine teleporting a 200 pound block directly into the same space already occupied by another 200 pound block. A rather impressive explosion should result as most of the chemical bonds in whatever used to be there are suddenly broken all at once. At the very least, it generates a shockwave as before, only with a large amount of shrapnel mixed in.



    3) Let's take his to the logical extreme: use simulations partial teleportations to compress an object down to a single point. Literally. Depending on how precise and accurate your teleporter can be, this should result in a nuclear explosion, as the an entire 200 pounds of atomic nuclei are forced into the same space, destabilizing them and causing them to break up. This should result in a nuclear explosion. Depending on material teleported, it may just result in irradiating the surrounding area with exotic particles (you might even want this outcome, kill the people, leave the buildings...). Alternatively, it also might form a black hole and destroy the Earth. Be careful with this one...



    As a disclaimer, I haven't run the numbers on any of these situations. They just seem like logical consequences based on my understanding of physics. I'm be happy to be corrected if someone else cares to do so.






    share|improve this answer









    $endgroup$













    • $begingroup$
      The only downside I see is that you have to be touching the teleported stuff. Presumably at both ends.
      $endgroup$
      – IT Alex
      12 hours ago










    • $begingroup$
      Ah, yes, that would make a difference. I was assuming you had to be touching it on the sending end, not the receiving end. Still, if there's no cool down, the teleporter might have time to jump away in time...?
      $endgroup$
      – jpgo5000
      11 hours ago



















    0












    $begingroup$

    Not an answer, but you want to check this story out. It's a Harry Potter fanfiction where Harry realizes that Apparation (teleportation for the HP world) is essentially the most important combat tool available in his world. He goes on to focus his prodigious skills into learning extremely fast, accurate, silent, and seemingly-randomized patterns of teleportation which acts as a massive force-multiplier in his battles.



    Definitely worth a read.



    https://www.fanfiction.net/s/7093738/1/Brutal-Harry






    share|improve this answer









    $endgroup$





















      0












      $begingroup$

      I don't know, that sounds overpowered and a bit silly. Teleportation with limits is much more interesting. Like not being able to teleport others out of danger, or not being able to teleport sufficiently far away or more than a few times in quick succession. In Project Horizons there was a character who would teleport behind someone, stab them with a sword, grab their gun and shoot their buddy, use them as a shield, teleport above another guy, crash stomp them, shoot another guy in the head, drop a grenade and teleport out the door and stroll away all while singing a happy song.






      share|improve this answer









      $endgroup$













        Your Answer





        StackExchange.ifUsing("editor", function () {
        return StackExchange.using("mathjaxEditing", function () {
        StackExchange.MarkdownEditor.creationCallbacks.add(function (editor, postfix) {
        StackExchange.mathjaxEditing.prepareWmdForMathJax(editor, postfix, [["$", "$"], ["\\(","\\)"]]);
        });
        });
        }, "mathjax-editing");

        StackExchange.ready(function() {
        var channelOptions = {
        tags: "".split(" "),
        id: "579"
        };
        initTagRenderer("".split(" "), "".split(" "), channelOptions);

        StackExchange.using("externalEditor", function() {
        // Have to fire editor after snippets, if snippets enabled
        if (StackExchange.settings.snippets.snippetsEnabled) {
        StackExchange.using("snippets", function() {
        createEditor();
        });
        }
        else {
        createEditor();
        }
        });

        function createEditor() {
        StackExchange.prepareEditor({
        heartbeatType: 'answer',
        autoActivateHeartbeat: false,
        convertImagesToLinks: false,
        noModals: true,
        showLowRepImageUploadWarning: true,
        reputationToPostImages: null,
        bindNavPrevention: true,
        postfix: "",
        imageUploader: {
        brandingHtml: "Powered by u003ca class="icon-imgur-white" href="https://imgur.com/"u003eu003c/au003e",
        contentPolicyHtml: "User contributions licensed under u003ca href="https://creativecommons.org/licenses/by-sa/3.0/"u003ecc by-sa 3.0 with attribution requiredu003c/au003e u003ca href="https://stackoverflow.com/legal/content-policy"u003e(content policy)u003c/au003e",
        allowUrls: true
        },
        noCode: true, onDemand: true,
        discardSelector: ".discard-answer"
        ,immediatelyShowMarkdownHelp:true
        });


        }
        });














        draft saved

        draft discarded


















        StackExchange.ready(
        function () {
        StackExchange.openid.initPostLogin('.new-post-login', 'https%3a%2f%2fworldbuilding.stackexchange.com%2fquestions%2f142436%2fpersonal-teleportation-as-a-weapon%23new-answer', 'question_page');
        }
        );

        Post as a guest















        Required, but never shown

























        11 Answers
        11






        active

        oldest

        votes








        11 Answers
        11






        active

        oldest

        votes









        active

        oldest

        votes






        active

        oldest

        votes









        14












        $begingroup$

        The following assumes that, like in most universes, you cannot simply teleport part of an object or person. That would be... too easy.



        Another assumption is that the power is "designed" to be safe for the user, so momentum, pressure etc. are adjusted reasonably well to the target frame of reference.



        The environment is your greatest weapon



        There aren't many threats that cannot be dealth with by teleporting them into mid-air, the deep sea, space, a sealed mine or a prison cell. Anything your character touches can be considered out of the fight. Sure, some of these locations are more exotic than others, but even if you want to keep the ability secret and cannot accept help from others, a sufficiently effective death trap can be improvised on a budget by e.g. renting a boat, buying a plane ticket or just crossing over a deep pit with a length of rope (that you remove afterwards).



        Conventional weapons and transportation work just as well



        Logistical limitations don't apply to you. Screw getting your own hands dirty, just provide an army of your choice with a steady stream of personnel, supplies and intel. That'll do more than a single person ever could.



        If you don't like other people risking their necks for your cause, get a fast vehicle (refueling at will means unlimited range) or at least heavy body armor for "lucky bullet" protection and stock up on heavy weapons. You don't need to reload, you can switch between weapons and angles of attack pretty much at will and no one can ever take any territory from you because the second you feel like it, they'll sit on a pile of armed bombs. Still too risky? Make contact once, then teleport home and port in a steady stream of drones.



        "Suicide" bombings are also an option, but I don't like those as much because you still need to get in range, and as we've already established, anything within touch range is doomed anyway. And anything a person could reach, a teleported missile or drone will have a much easier time with.



        Never underestimate gravity



        Assuming you're




        • ok at math

        • able to retain momentum when teleporting

        • able to afford a handful of trips to orbit


        nowhere is really safe from orbital bombardment. You'll have to experiment a bit to work out what types of ammunition work best but if you want to flatten a city, that's probably the way to go.






        share|improve this answer











        $endgroup$













        • $begingroup$
          If you can teleport to where you've been with the momentum you had last time you were there, getting to orbit is incrementally is reasonably cheap.
          $endgroup$
          – Joshua
          12 hours ago










        • $begingroup$
          "That would be... to easy." Indeed.
          $endgroup$
          – reirab
          11 hours ago












        • $begingroup$
          To add to your list of environmental factors, teleporting someone into the environment - i.e. half way into a wall or directly into the middle of a block of concrete would be pretty effective as well. Or just teleport half of someone to somewhere else.
          $endgroup$
          – Aaron Lavers
          5 hours ago


















        14












        $begingroup$

        The following assumes that, like in most universes, you cannot simply teleport part of an object or person. That would be... too easy.



        Another assumption is that the power is "designed" to be safe for the user, so momentum, pressure etc. are adjusted reasonably well to the target frame of reference.



        The environment is your greatest weapon



        There aren't many threats that cannot be dealth with by teleporting them into mid-air, the deep sea, space, a sealed mine or a prison cell. Anything your character touches can be considered out of the fight. Sure, some of these locations are more exotic than others, but even if you want to keep the ability secret and cannot accept help from others, a sufficiently effective death trap can be improvised on a budget by e.g. renting a boat, buying a plane ticket or just crossing over a deep pit with a length of rope (that you remove afterwards).



        Conventional weapons and transportation work just as well



        Logistical limitations don't apply to you. Screw getting your own hands dirty, just provide an army of your choice with a steady stream of personnel, supplies and intel. That'll do more than a single person ever could.



        If you don't like other people risking their necks for your cause, get a fast vehicle (refueling at will means unlimited range) or at least heavy body armor for "lucky bullet" protection and stock up on heavy weapons. You don't need to reload, you can switch between weapons and angles of attack pretty much at will and no one can ever take any territory from you because the second you feel like it, they'll sit on a pile of armed bombs. Still too risky? Make contact once, then teleport home and port in a steady stream of drones.



        "Suicide" bombings are also an option, but I don't like those as much because you still need to get in range, and as we've already established, anything within touch range is doomed anyway. And anything a person could reach, a teleported missile or drone will have a much easier time with.



        Never underestimate gravity



        Assuming you're




        • ok at math

        • able to retain momentum when teleporting

        • able to afford a handful of trips to orbit


        nowhere is really safe from orbital bombardment. You'll have to experiment a bit to work out what types of ammunition work best but if you want to flatten a city, that's probably the way to go.






        share|improve this answer











        $endgroup$













        • $begingroup$
          If you can teleport to where you've been with the momentum you had last time you were there, getting to orbit is incrementally is reasonably cheap.
          $endgroup$
          – Joshua
          12 hours ago










        • $begingroup$
          "That would be... to easy." Indeed.
          $endgroup$
          – reirab
          11 hours ago












        • $begingroup$
          To add to your list of environmental factors, teleporting someone into the environment - i.e. half way into a wall or directly into the middle of a block of concrete would be pretty effective as well. Or just teleport half of someone to somewhere else.
          $endgroup$
          – Aaron Lavers
          5 hours ago
















        14












        14








        14





        $begingroup$

        The following assumes that, like in most universes, you cannot simply teleport part of an object or person. That would be... too easy.



        Another assumption is that the power is "designed" to be safe for the user, so momentum, pressure etc. are adjusted reasonably well to the target frame of reference.



        The environment is your greatest weapon



        There aren't many threats that cannot be dealth with by teleporting them into mid-air, the deep sea, space, a sealed mine or a prison cell. Anything your character touches can be considered out of the fight. Sure, some of these locations are more exotic than others, but even if you want to keep the ability secret and cannot accept help from others, a sufficiently effective death trap can be improvised on a budget by e.g. renting a boat, buying a plane ticket or just crossing over a deep pit with a length of rope (that you remove afterwards).



        Conventional weapons and transportation work just as well



        Logistical limitations don't apply to you. Screw getting your own hands dirty, just provide an army of your choice with a steady stream of personnel, supplies and intel. That'll do more than a single person ever could.



        If you don't like other people risking their necks for your cause, get a fast vehicle (refueling at will means unlimited range) or at least heavy body armor for "lucky bullet" protection and stock up on heavy weapons. You don't need to reload, you can switch between weapons and angles of attack pretty much at will and no one can ever take any territory from you because the second you feel like it, they'll sit on a pile of armed bombs. Still too risky? Make contact once, then teleport home and port in a steady stream of drones.



        "Suicide" bombings are also an option, but I don't like those as much because you still need to get in range, and as we've already established, anything within touch range is doomed anyway. And anything a person could reach, a teleported missile or drone will have a much easier time with.



        Never underestimate gravity



        Assuming you're




        • ok at math

        • able to retain momentum when teleporting

        • able to afford a handful of trips to orbit


        nowhere is really safe from orbital bombardment. You'll have to experiment a bit to work out what types of ammunition work best but if you want to flatten a city, that's probably the way to go.






        share|improve this answer











        $endgroup$



        The following assumes that, like in most universes, you cannot simply teleport part of an object or person. That would be... too easy.



        Another assumption is that the power is "designed" to be safe for the user, so momentum, pressure etc. are adjusted reasonably well to the target frame of reference.



        The environment is your greatest weapon



        There aren't many threats that cannot be dealth with by teleporting them into mid-air, the deep sea, space, a sealed mine or a prison cell. Anything your character touches can be considered out of the fight. Sure, some of these locations are more exotic than others, but even if you want to keep the ability secret and cannot accept help from others, a sufficiently effective death trap can be improvised on a budget by e.g. renting a boat, buying a plane ticket or just crossing over a deep pit with a length of rope (that you remove afterwards).



        Conventional weapons and transportation work just as well



        Logistical limitations don't apply to you. Screw getting your own hands dirty, just provide an army of your choice with a steady stream of personnel, supplies and intel. That'll do more than a single person ever could.



        If you don't like other people risking their necks for your cause, get a fast vehicle (refueling at will means unlimited range) or at least heavy body armor for "lucky bullet" protection and stock up on heavy weapons. You don't need to reload, you can switch between weapons and angles of attack pretty much at will and no one can ever take any territory from you because the second you feel like it, they'll sit on a pile of armed bombs. Still too risky? Make contact once, then teleport home and port in a steady stream of drones.



        "Suicide" bombings are also an option, but I don't like those as much because you still need to get in range, and as we've already established, anything within touch range is doomed anyway. And anything a person could reach, a teleported missile or drone will have a much easier time with.



        Never underestimate gravity



        Assuming you're




        • ok at math

        • able to retain momentum when teleporting

        • able to afford a handful of trips to orbit


        nowhere is really safe from orbital bombardment. You'll have to experiment a bit to work out what types of ammunition work best but if you want to flatten a city, that's probably the way to go.







        share|improve this answer














        share|improve this answer



        share|improve this answer








        edited 15 hours ago

























        answered 15 hours ago









        Ruther RendommeleighRuther Rendommeleigh

        493110




        493110












        • $begingroup$
          If you can teleport to where you've been with the momentum you had last time you were there, getting to orbit is incrementally is reasonably cheap.
          $endgroup$
          – Joshua
          12 hours ago










        • $begingroup$
          "That would be... to easy." Indeed.
          $endgroup$
          – reirab
          11 hours ago












        • $begingroup$
          To add to your list of environmental factors, teleporting someone into the environment - i.e. half way into a wall or directly into the middle of a block of concrete would be pretty effective as well. Or just teleport half of someone to somewhere else.
          $endgroup$
          – Aaron Lavers
          5 hours ago




















        • $begingroup$
          If you can teleport to where you've been with the momentum you had last time you were there, getting to orbit is incrementally is reasonably cheap.
          $endgroup$
          – Joshua
          12 hours ago










        • $begingroup$
          "That would be... to easy." Indeed.
          $endgroup$
          – reirab
          11 hours ago












        • $begingroup$
          To add to your list of environmental factors, teleporting someone into the environment - i.e. half way into a wall or directly into the middle of a block of concrete would be pretty effective as well. Or just teleport half of someone to somewhere else.
          $endgroup$
          – Aaron Lavers
          5 hours ago


















        $begingroup$
        If you can teleport to where you've been with the momentum you had last time you were there, getting to orbit is incrementally is reasonably cheap.
        $endgroup$
        – Joshua
        12 hours ago




        $begingroup$
        If you can teleport to where you've been with the momentum you had last time you were there, getting to orbit is incrementally is reasonably cheap.
        $endgroup$
        – Joshua
        12 hours ago












        $begingroup$
        "That would be... to easy." Indeed.
        $endgroup$
        – reirab
        11 hours ago






        $begingroup$
        "That would be... to easy." Indeed.
        $endgroup$
        – reirab
        11 hours ago














        $begingroup$
        To add to your list of environmental factors, teleporting someone into the environment - i.e. half way into a wall or directly into the middle of a block of concrete would be pretty effective as well. Or just teleport half of someone to somewhere else.
        $endgroup$
        – Aaron Lavers
        5 hours ago






        $begingroup$
        To add to your list of environmental factors, teleporting someone into the environment - i.e. half way into a wall or directly into the middle of a block of concrete would be pretty effective as well. Or just teleport half of someone to somewhere else.
        $endgroup$
        – Aaron Lavers
        5 hours ago













        7












        $begingroup$

        So like Friendlysociopath used, I will be borrowing from anime to answer.



        A Certain Scientific Teleporter



        In the series A Certain Magical Index and A Certain Scientific Railgun we're introduced to a character who can teleport. They're a part of a neighborhood watch-like program (so they don't go about killing people) but they have an unusually high combat incident count. Their main line of attack is to teleport metal darts into the clothes of opponents and pin them to surfaces. Usually they knock them against the surfaces first (satisfying a more narrow application of the "Must have visited this place before" clause). But they have a brevity of application to their powers.




        1. They can teleport objects into people. So if you're a teleporter looking to just end any 1v1 engagement, teleport a needle into their heart or brain. Done. Assuming the "visited this place before" means only that you've physically seen it. But if we're more specific (e.g. you've had to have touched the place before") then we still have the standard option of restraining people with heavy objects once you've touched them.


        2. They can teleport material into structural supports and bring down buildings. Again, assuming you just have to physically have seen your target place before then you can just teleport glass panes from a building's windows to their structural supports and down goes a couple hundred million dollars not to mention the mass panic. Although, this involves changing the orientation of an object in teleportation. Another limitation. But if you have to touch all of the target you're teleporting to then bringing down buildings is a little out of reach. However, you can still cause mass panic because...



        3. You can teleport double your mass to the tops of buildings and have them come crashing down. 140kg dropping 20 stories will hurt a great deal. Again, if you only have to have seen the target, dropping 140kg asphalt balls from the cloud line will be incredibly damaging depending on cooldown times or other physical limitations. The character in question doesn't do this but a Rival teleporter does try to do this to crush the character.







        So you have options in combat when it comes to being a teleporter. As others have said you can really abuse the rule and win a lot of fights just by sending an opponent free solo skydiving or jamming their own weapon into their brain. On a less lethal note, dropping 140kg's of sand on someone will buy you time to incapacitate them or kill them. Teleporting 140kg's of industrial staples will pin someone to a surface or fewer metal stakes will do the same. Even just threatening teleporting a needle into someone's brain will stop a lot of fights.



        Against groups, like IT Alex stated, teleport them away till you get down to a manageable number then start incapacitating or killing as you see fit. If you have to fight full groups, hope for the environment to be in your favor and teleport holes into the ground in front of people and use that dirt to cover someone. Or start throwing javelins at people.



        And if you're just in it for the mass panic, you have too many options to count. You can't blow up cities but you can royally mess with them. Teleport small pieces of bridge supports away, drop asphalt boulders from on high, ruin electrical substations, even destroy buildings. You can cause quite a bit of damage in a very short amount of time with little effort.






        share|improve this answer









        $endgroup$













        • $begingroup$
          Since you are assuming they don't need to see where they are teleporting, I'd also add teleporting something into the ground below a structure or into the infrastructure, such as natural gas or electrical lines. Anything sufficiently large will cause a massive explosion of earth, besides the disruption of the services.
          $endgroup$
          – computercarguy
          10 hours ago










        • $begingroup$
          The crux really lies on how you apply "I've visited this place in person before". This is why I gave two distinct ways to deal with problems. If we speak conservatively and you have to have touched the object and the target is a lot different than just having to have seen the target in the past. In this case, if we're being conservative about mass destruction, you can't really teleport something into the ground far enough for it to hit infrastructure (maybe artillery drop it). But liberally you can. It all depends on the rules.
          $endgroup$
          – GuidingOlive
          10 hours ago












        • $begingroup$
          Even if we aren't using mass destruction, simply a large mass appearing inside another mass will cause all kinds of stresses that will react similar to an explosion. The harder the material the better, in this case, since a sandy ground might just deform some, but even sufficiently hard limestone should rupture quite significantly. And yes, this would be liberal with where something can be transported, such as the center of a solid object.
          $endgroup$
          – computercarguy
          10 hours ago
















        7












        $begingroup$

        So like Friendlysociopath used, I will be borrowing from anime to answer.



        A Certain Scientific Teleporter



        In the series A Certain Magical Index and A Certain Scientific Railgun we're introduced to a character who can teleport. They're a part of a neighborhood watch-like program (so they don't go about killing people) but they have an unusually high combat incident count. Their main line of attack is to teleport metal darts into the clothes of opponents and pin them to surfaces. Usually they knock them against the surfaces first (satisfying a more narrow application of the "Must have visited this place before" clause). But they have a brevity of application to their powers.




        1. They can teleport objects into people. So if you're a teleporter looking to just end any 1v1 engagement, teleport a needle into their heart or brain. Done. Assuming the "visited this place before" means only that you've physically seen it. But if we're more specific (e.g. you've had to have touched the place before") then we still have the standard option of restraining people with heavy objects once you've touched them.


        2. They can teleport material into structural supports and bring down buildings. Again, assuming you just have to physically have seen your target place before then you can just teleport glass panes from a building's windows to their structural supports and down goes a couple hundred million dollars not to mention the mass panic. Although, this involves changing the orientation of an object in teleportation. Another limitation. But if you have to touch all of the target you're teleporting to then bringing down buildings is a little out of reach. However, you can still cause mass panic because...



        3. You can teleport double your mass to the tops of buildings and have them come crashing down. 140kg dropping 20 stories will hurt a great deal. Again, if you only have to have seen the target, dropping 140kg asphalt balls from the cloud line will be incredibly damaging depending on cooldown times or other physical limitations. The character in question doesn't do this but a Rival teleporter does try to do this to crush the character.







        So you have options in combat when it comes to being a teleporter. As others have said you can really abuse the rule and win a lot of fights just by sending an opponent free solo skydiving or jamming their own weapon into their brain. On a less lethal note, dropping 140kg's of sand on someone will buy you time to incapacitate them or kill them. Teleporting 140kg's of industrial staples will pin someone to a surface or fewer metal stakes will do the same. Even just threatening teleporting a needle into someone's brain will stop a lot of fights.



        Against groups, like IT Alex stated, teleport them away till you get down to a manageable number then start incapacitating or killing as you see fit. If you have to fight full groups, hope for the environment to be in your favor and teleport holes into the ground in front of people and use that dirt to cover someone. Or start throwing javelins at people.



        And if you're just in it for the mass panic, you have too many options to count. You can't blow up cities but you can royally mess with them. Teleport small pieces of bridge supports away, drop asphalt boulders from on high, ruin electrical substations, even destroy buildings. You can cause quite a bit of damage in a very short amount of time with little effort.






        share|improve this answer









        $endgroup$













        • $begingroup$
          Since you are assuming they don't need to see where they are teleporting, I'd also add teleporting something into the ground below a structure or into the infrastructure, such as natural gas or electrical lines. Anything sufficiently large will cause a massive explosion of earth, besides the disruption of the services.
          $endgroup$
          – computercarguy
          10 hours ago










        • $begingroup$
          The crux really lies on how you apply "I've visited this place in person before". This is why I gave two distinct ways to deal with problems. If we speak conservatively and you have to have touched the object and the target is a lot different than just having to have seen the target in the past. In this case, if we're being conservative about mass destruction, you can't really teleport something into the ground far enough for it to hit infrastructure (maybe artillery drop it). But liberally you can. It all depends on the rules.
          $endgroup$
          – GuidingOlive
          10 hours ago












        • $begingroup$
          Even if we aren't using mass destruction, simply a large mass appearing inside another mass will cause all kinds of stresses that will react similar to an explosion. The harder the material the better, in this case, since a sandy ground might just deform some, but even sufficiently hard limestone should rupture quite significantly. And yes, this would be liberal with where something can be transported, such as the center of a solid object.
          $endgroup$
          – computercarguy
          10 hours ago














        7












        7








        7





        $begingroup$

        So like Friendlysociopath used, I will be borrowing from anime to answer.



        A Certain Scientific Teleporter



        In the series A Certain Magical Index and A Certain Scientific Railgun we're introduced to a character who can teleport. They're a part of a neighborhood watch-like program (so they don't go about killing people) but they have an unusually high combat incident count. Their main line of attack is to teleport metal darts into the clothes of opponents and pin them to surfaces. Usually they knock them against the surfaces first (satisfying a more narrow application of the "Must have visited this place before" clause). But they have a brevity of application to their powers.




        1. They can teleport objects into people. So if you're a teleporter looking to just end any 1v1 engagement, teleport a needle into their heart or brain. Done. Assuming the "visited this place before" means only that you've physically seen it. But if we're more specific (e.g. you've had to have touched the place before") then we still have the standard option of restraining people with heavy objects once you've touched them.


        2. They can teleport material into structural supports and bring down buildings. Again, assuming you just have to physically have seen your target place before then you can just teleport glass panes from a building's windows to their structural supports and down goes a couple hundred million dollars not to mention the mass panic. Although, this involves changing the orientation of an object in teleportation. Another limitation. But if you have to touch all of the target you're teleporting to then bringing down buildings is a little out of reach. However, you can still cause mass panic because...



        3. You can teleport double your mass to the tops of buildings and have them come crashing down. 140kg dropping 20 stories will hurt a great deal. Again, if you only have to have seen the target, dropping 140kg asphalt balls from the cloud line will be incredibly damaging depending on cooldown times or other physical limitations. The character in question doesn't do this but a Rival teleporter does try to do this to crush the character.







        So you have options in combat when it comes to being a teleporter. As others have said you can really abuse the rule and win a lot of fights just by sending an opponent free solo skydiving or jamming their own weapon into their brain. On a less lethal note, dropping 140kg's of sand on someone will buy you time to incapacitate them or kill them. Teleporting 140kg's of industrial staples will pin someone to a surface or fewer metal stakes will do the same. Even just threatening teleporting a needle into someone's brain will stop a lot of fights.



        Against groups, like IT Alex stated, teleport them away till you get down to a manageable number then start incapacitating or killing as you see fit. If you have to fight full groups, hope for the environment to be in your favor and teleport holes into the ground in front of people and use that dirt to cover someone. Or start throwing javelins at people.



        And if you're just in it for the mass panic, you have too many options to count. You can't blow up cities but you can royally mess with them. Teleport small pieces of bridge supports away, drop asphalt boulders from on high, ruin electrical substations, even destroy buildings. You can cause quite a bit of damage in a very short amount of time with little effort.






        share|improve this answer









        $endgroup$



        So like Friendlysociopath used, I will be borrowing from anime to answer.



        A Certain Scientific Teleporter



        In the series A Certain Magical Index and A Certain Scientific Railgun we're introduced to a character who can teleport. They're a part of a neighborhood watch-like program (so they don't go about killing people) but they have an unusually high combat incident count. Their main line of attack is to teleport metal darts into the clothes of opponents and pin them to surfaces. Usually they knock them against the surfaces first (satisfying a more narrow application of the "Must have visited this place before" clause). But they have a brevity of application to their powers.




        1. They can teleport objects into people. So if you're a teleporter looking to just end any 1v1 engagement, teleport a needle into their heart or brain. Done. Assuming the "visited this place before" means only that you've physically seen it. But if we're more specific (e.g. you've had to have touched the place before") then we still have the standard option of restraining people with heavy objects once you've touched them.


        2. They can teleport material into structural supports and bring down buildings. Again, assuming you just have to physically have seen your target place before then you can just teleport glass panes from a building's windows to their structural supports and down goes a couple hundred million dollars not to mention the mass panic. Although, this involves changing the orientation of an object in teleportation. Another limitation. But if you have to touch all of the target you're teleporting to then bringing down buildings is a little out of reach. However, you can still cause mass panic because...



        3. You can teleport double your mass to the tops of buildings and have them come crashing down. 140kg dropping 20 stories will hurt a great deal. Again, if you only have to have seen the target, dropping 140kg asphalt balls from the cloud line will be incredibly damaging depending on cooldown times or other physical limitations. The character in question doesn't do this but a Rival teleporter does try to do this to crush the character.







        So you have options in combat when it comes to being a teleporter. As others have said you can really abuse the rule and win a lot of fights just by sending an opponent free solo skydiving or jamming their own weapon into their brain. On a less lethal note, dropping 140kg's of sand on someone will buy you time to incapacitate them or kill them. Teleporting 140kg's of industrial staples will pin someone to a surface or fewer metal stakes will do the same. Even just threatening teleporting a needle into someone's brain will stop a lot of fights.



        Against groups, like IT Alex stated, teleport them away till you get down to a manageable number then start incapacitating or killing as you see fit. If you have to fight full groups, hope for the environment to be in your favor and teleport holes into the ground in front of people and use that dirt to cover someone. Or start throwing javelins at people.



        And if you're just in it for the mass panic, you have too many options to count. You can't blow up cities but you can royally mess with them. Teleport small pieces of bridge supports away, drop asphalt boulders from on high, ruin electrical substations, even destroy buildings. You can cause quite a bit of damage in a very short amount of time with little effort.







        share|improve this answer












        share|improve this answer



        share|improve this answer










        answered 13 hours ago









        GuidingOliveGuidingOlive

        6131214




        6131214












        • $begingroup$
          Since you are assuming they don't need to see where they are teleporting, I'd also add teleporting something into the ground below a structure or into the infrastructure, such as natural gas or electrical lines. Anything sufficiently large will cause a massive explosion of earth, besides the disruption of the services.
          $endgroup$
          – computercarguy
          10 hours ago










        • $begingroup$
          The crux really lies on how you apply "I've visited this place in person before". This is why I gave two distinct ways to deal with problems. If we speak conservatively and you have to have touched the object and the target is a lot different than just having to have seen the target in the past. In this case, if we're being conservative about mass destruction, you can't really teleport something into the ground far enough for it to hit infrastructure (maybe artillery drop it). But liberally you can. It all depends on the rules.
          $endgroup$
          – GuidingOlive
          10 hours ago












        • $begingroup$
          Even if we aren't using mass destruction, simply a large mass appearing inside another mass will cause all kinds of stresses that will react similar to an explosion. The harder the material the better, in this case, since a sandy ground might just deform some, but even sufficiently hard limestone should rupture quite significantly. And yes, this would be liberal with where something can be transported, such as the center of a solid object.
          $endgroup$
          – computercarguy
          10 hours ago


















        • $begingroup$
          Since you are assuming they don't need to see where they are teleporting, I'd also add teleporting something into the ground below a structure or into the infrastructure, such as natural gas or electrical lines. Anything sufficiently large will cause a massive explosion of earth, besides the disruption of the services.
          $endgroup$
          – computercarguy
          10 hours ago










        • $begingroup$
          The crux really lies on how you apply "I've visited this place in person before". This is why I gave two distinct ways to deal with problems. If we speak conservatively and you have to have touched the object and the target is a lot different than just having to have seen the target in the past. In this case, if we're being conservative about mass destruction, you can't really teleport something into the ground far enough for it to hit infrastructure (maybe artillery drop it). But liberally you can. It all depends on the rules.
          $endgroup$
          – GuidingOlive
          10 hours ago












        • $begingroup$
          Even if we aren't using mass destruction, simply a large mass appearing inside another mass will cause all kinds of stresses that will react similar to an explosion. The harder the material the better, in this case, since a sandy ground might just deform some, but even sufficiently hard limestone should rupture quite significantly. And yes, this would be liberal with where something can be transported, such as the center of a solid object.
          $endgroup$
          – computercarguy
          10 hours ago
















        $begingroup$
        Since you are assuming they don't need to see where they are teleporting, I'd also add teleporting something into the ground below a structure or into the infrastructure, such as natural gas or electrical lines. Anything sufficiently large will cause a massive explosion of earth, besides the disruption of the services.
        $endgroup$
        – computercarguy
        10 hours ago




        $begingroup$
        Since you are assuming they don't need to see where they are teleporting, I'd also add teleporting something into the ground below a structure or into the infrastructure, such as natural gas or electrical lines. Anything sufficiently large will cause a massive explosion of earth, besides the disruption of the services.
        $endgroup$
        – computercarguy
        10 hours ago












        $begingroup$
        The crux really lies on how you apply "I've visited this place in person before". This is why I gave two distinct ways to deal with problems. If we speak conservatively and you have to have touched the object and the target is a lot different than just having to have seen the target in the past. In this case, if we're being conservative about mass destruction, you can't really teleport something into the ground far enough for it to hit infrastructure (maybe artillery drop it). But liberally you can. It all depends on the rules.
        $endgroup$
        – GuidingOlive
        10 hours ago






        $begingroup$
        The crux really lies on how you apply "I've visited this place in person before". This is why I gave two distinct ways to deal with problems. If we speak conservatively and you have to have touched the object and the target is a lot different than just having to have seen the target in the past. In this case, if we're being conservative about mass destruction, you can't really teleport something into the ground far enough for it to hit infrastructure (maybe artillery drop it). But liberally you can. It all depends on the rules.
        $endgroup$
        – GuidingOlive
        10 hours ago














        $begingroup$
        Even if we aren't using mass destruction, simply a large mass appearing inside another mass will cause all kinds of stresses that will react similar to an explosion. The harder the material the better, in this case, since a sandy ground might just deform some, but even sufficiently hard limestone should rupture quite significantly. And yes, this would be liberal with where something can be transported, such as the center of a solid object.
        $endgroup$
        – computercarguy
        10 hours ago




        $begingroup$
        Even if we aren't using mass destruction, simply a large mass appearing inside another mass will cause all kinds of stresses that will react similar to an explosion. The harder the material the better, in this case, since a sandy ground might just deform some, but even sufficiently hard limestone should rupture quite significantly. And yes, this would be liberal with where something can be transported, such as the center of a solid object.
        $endgroup$
        – computercarguy
        10 hours ago











        6












        $begingroup$


        The Teleporter can teleport themselves and/or any object (including people) they are touching, with a total mass limit of roughly double their own mass
        People/things/self can only be teleported to places which the Teleporter has previously visited in person




        If you've ever watched Darker Than Black the protagonist faces someone with a teleportation power. Except instead of teleporting themselves they teleport a fist-sized bunch of matter- which they typically use to swap pieces of building with the heart of the person they're trying to assassinate.






        share|improve this answer









        $endgroup$













        • $begingroup$
          "People/things/self can only be teleported to places which the Teleporter has previously visited in person" -- I'm pretty sure the Teleporter has not physically been inside their intended victim before, so swapping out internal organs (or even swapping things in) isn't going to happen I don't think.
          $endgroup$
          – Doktor J
          10 hours ago
















        6












        $begingroup$


        The Teleporter can teleport themselves and/or any object (including people) they are touching, with a total mass limit of roughly double their own mass
        People/things/self can only be teleported to places which the Teleporter has previously visited in person




        If you've ever watched Darker Than Black the protagonist faces someone with a teleportation power. Except instead of teleporting themselves they teleport a fist-sized bunch of matter- which they typically use to swap pieces of building with the heart of the person they're trying to assassinate.






        share|improve this answer









        $endgroup$













        • $begingroup$
          "People/things/self can only be teleported to places which the Teleporter has previously visited in person" -- I'm pretty sure the Teleporter has not physically been inside their intended victim before, so swapping out internal organs (or even swapping things in) isn't going to happen I don't think.
          $endgroup$
          – Doktor J
          10 hours ago














        6












        6








        6





        $begingroup$


        The Teleporter can teleport themselves and/or any object (including people) they are touching, with a total mass limit of roughly double their own mass
        People/things/self can only be teleported to places which the Teleporter has previously visited in person




        If you've ever watched Darker Than Black the protagonist faces someone with a teleportation power. Except instead of teleporting themselves they teleport a fist-sized bunch of matter- which they typically use to swap pieces of building with the heart of the person they're trying to assassinate.






        share|improve this answer









        $endgroup$




        The Teleporter can teleport themselves and/or any object (including people) they are touching, with a total mass limit of roughly double their own mass
        People/things/self can only be teleported to places which the Teleporter has previously visited in person




        If you've ever watched Darker Than Black the protagonist faces someone with a teleportation power. Except instead of teleporting themselves they teleport a fist-sized bunch of matter- which they typically use to swap pieces of building with the heart of the person they're trying to assassinate.







        share|improve this answer












        share|improve this answer



        share|improve this answer










        answered 16 hours ago









        FriendlysociopathFriendlysociopath

        1,2021613




        1,2021613












        • $begingroup$
          "People/things/self can only be teleported to places which the Teleporter has previously visited in person" -- I'm pretty sure the Teleporter has not physically been inside their intended victim before, so swapping out internal organs (or even swapping things in) isn't going to happen I don't think.
          $endgroup$
          – Doktor J
          10 hours ago


















        • $begingroup$
          "People/things/self can only be teleported to places which the Teleporter has previously visited in person" -- I'm pretty sure the Teleporter has not physically been inside their intended victim before, so swapping out internal organs (or even swapping things in) isn't going to happen I don't think.
          $endgroup$
          – Doktor J
          10 hours ago
















        $begingroup$
        "People/things/self can only be teleported to places which the Teleporter has previously visited in person" -- I'm pretty sure the Teleporter has not physically been inside their intended victim before, so swapping out internal organs (or even swapping things in) isn't going to happen I don't think.
        $endgroup$
        – Doktor J
        10 hours ago




        $begingroup$
        "People/things/self can only be teleported to places which the Teleporter has previously visited in person" -- I'm pretty sure the Teleporter has not physically been inside their intended victim before, so swapping out internal organs (or even swapping things in) isn't going to happen I don't think.
        $endgroup$
        – Doktor J
        10 hours ago











        5












        $begingroup$

        Air Drop



        In X-men: First Class, a guy with Nightcrawler's powers kills people by teleporting into the air with them and then dropping them while he teleports back down. Since he can teleport right up to someone to grab them, there isn't much defense against this. You specify only places he has been, but as long as he's gone skydiving even once, this shouldn't be a problem.



        (Others mentioned that specific attack in the comments section. I just noticed, even thought they posted before I did. Credit should go to @Willk and @BKLassen. Or, should I just delete that part?)



        Partial Teleport



        If your teleporter doesn't need to take the whole object or person with him, he could put his hand on a guy, and teleport away with only part of them. Instant decapitations, or 'Disarming'.






        share|improve this answer











        $endgroup$













        • $begingroup$
          I'm just considering the implications of a Nightcrawler-like character who has gone skydiving exactly once and continues to use that strategy every time he's in a fight... The number of bodies found piled up in that one spot would start to get a little suspicious...
          $endgroup$
          – Darrel Hoffman
          13 hours ago






        • 1




          $begingroup$
          All the person has to do is take a flight across the Atlantic or Pacific. Heck, even a cruise ship will do but it's not as certain. Then they can teleport themselves and the target somewhere where no one will find the body. With the airline flight, the teleporter would be so high that he would have to deal with very low airpressure and extreme cold for a fraction of a second. Off of a cruise ship, the kill isn't certain since they arrive closer to the water and in a shipping lane.
          $endgroup$
          – ShadoCat
          13 hours ago






        • 2




          $begingroup$
          The Earth is rotating and orbiting the Sun, and the Milky Way (with the Earth and the Sun in it) is also rotating. Given the density of space, the location he has been to is now with very high probability in outer space, and humans can't survive the cold and the lack of air for more than a minute. This is practical for taking someone to outer space, and thus quickly killing them, but then teleporting back to a safe place will be a problem.
          $endgroup$
          – pts
          13 hours ago








        • 1




          $begingroup$
          @pts Currently we have no absolute reference frame, that is, no point in space which is absolutely "not moving". (We may find one in the future, but probably not.) So we only have relative reference frames to choose from. At the very least, Teleporters will teleport with reference to the Earth, if not with reference to their destination e.g. a moving vehicle.
          $endgroup$
          – Martin Carney
          12 hours ago
















        5












        $begingroup$

        Air Drop



        In X-men: First Class, a guy with Nightcrawler's powers kills people by teleporting into the air with them and then dropping them while he teleports back down. Since he can teleport right up to someone to grab them, there isn't much defense against this. You specify only places he has been, but as long as he's gone skydiving even once, this shouldn't be a problem.



        (Others mentioned that specific attack in the comments section. I just noticed, even thought they posted before I did. Credit should go to @Willk and @BKLassen. Or, should I just delete that part?)



        Partial Teleport



        If your teleporter doesn't need to take the whole object or person with him, he could put his hand on a guy, and teleport away with only part of them. Instant decapitations, or 'Disarming'.






        share|improve this answer











        $endgroup$













        • $begingroup$
          I'm just considering the implications of a Nightcrawler-like character who has gone skydiving exactly once and continues to use that strategy every time he's in a fight... The number of bodies found piled up in that one spot would start to get a little suspicious...
          $endgroup$
          – Darrel Hoffman
          13 hours ago






        • 1




          $begingroup$
          All the person has to do is take a flight across the Atlantic or Pacific. Heck, even a cruise ship will do but it's not as certain. Then they can teleport themselves and the target somewhere where no one will find the body. With the airline flight, the teleporter would be so high that he would have to deal with very low airpressure and extreme cold for a fraction of a second. Off of a cruise ship, the kill isn't certain since they arrive closer to the water and in a shipping lane.
          $endgroup$
          – ShadoCat
          13 hours ago






        • 2




          $begingroup$
          The Earth is rotating and orbiting the Sun, and the Milky Way (with the Earth and the Sun in it) is also rotating. Given the density of space, the location he has been to is now with very high probability in outer space, and humans can't survive the cold and the lack of air for more than a minute. This is practical for taking someone to outer space, and thus quickly killing them, but then teleporting back to a safe place will be a problem.
          $endgroup$
          – pts
          13 hours ago








        • 1




          $begingroup$
          @pts Currently we have no absolute reference frame, that is, no point in space which is absolutely "not moving". (We may find one in the future, but probably not.) So we only have relative reference frames to choose from. At the very least, Teleporters will teleport with reference to the Earth, if not with reference to their destination e.g. a moving vehicle.
          $endgroup$
          – Martin Carney
          12 hours ago














        5












        5








        5





        $begingroup$

        Air Drop



        In X-men: First Class, a guy with Nightcrawler's powers kills people by teleporting into the air with them and then dropping them while he teleports back down. Since he can teleport right up to someone to grab them, there isn't much defense against this. You specify only places he has been, but as long as he's gone skydiving even once, this shouldn't be a problem.



        (Others mentioned that specific attack in the comments section. I just noticed, even thought they posted before I did. Credit should go to @Willk and @BKLassen. Or, should I just delete that part?)



        Partial Teleport



        If your teleporter doesn't need to take the whole object or person with him, he could put his hand on a guy, and teleport away with only part of them. Instant decapitations, or 'Disarming'.






        share|improve this answer











        $endgroup$



        Air Drop



        In X-men: First Class, a guy with Nightcrawler's powers kills people by teleporting into the air with them and then dropping them while he teleports back down. Since he can teleport right up to someone to grab them, there isn't much defense against this. You specify only places he has been, but as long as he's gone skydiving even once, this shouldn't be a problem.



        (Others mentioned that specific attack in the comments section. I just noticed, even thought they posted before I did. Credit should go to @Willk and @BKLassen. Or, should I just delete that part?)



        Partial Teleport



        If your teleporter doesn't need to take the whole object or person with him, he could put his hand on a guy, and teleport away with only part of them. Instant decapitations, or 'Disarming'.







        share|improve this answer














        share|improve this answer



        share|improve this answer








        edited 15 hours ago

























        answered 15 hours ago









        Divided By OneDivided By One

        1664




        1664












        • $begingroup$
          I'm just considering the implications of a Nightcrawler-like character who has gone skydiving exactly once and continues to use that strategy every time he's in a fight... The number of bodies found piled up in that one spot would start to get a little suspicious...
          $endgroup$
          – Darrel Hoffman
          13 hours ago






        • 1




          $begingroup$
          All the person has to do is take a flight across the Atlantic or Pacific. Heck, even a cruise ship will do but it's not as certain. Then they can teleport themselves and the target somewhere where no one will find the body. With the airline flight, the teleporter would be so high that he would have to deal with very low airpressure and extreme cold for a fraction of a second. Off of a cruise ship, the kill isn't certain since they arrive closer to the water and in a shipping lane.
          $endgroup$
          – ShadoCat
          13 hours ago






        • 2




          $begingroup$
          The Earth is rotating and orbiting the Sun, and the Milky Way (with the Earth and the Sun in it) is also rotating. Given the density of space, the location he has been to is now with very high probability in outer space, and humans can't survive the cold and the lack of air for more than a minute. This is practical for taking someone to outer space, and thus quickly killing them, but then teleporting back to a safe place will be a problem.
          $endgroup$
          – pts
          13 hours ago








        • 1




          $begingroup$
          @pts Currently we have no absolute reference frame, that is, no point in space which is absolutely "not moving". (We may find one in the future, but probably not.) So we only have relative reference frames to choose from. At the very least, Teleporters will teleport with reference to the Earth, if not with reference to their destination e.g. a moving vehicle.
          $endgroup$
          – Martin Carney
          12 hours ago


















        • $begingroup$
          I'm just considering the implications of a Nightcrawler-like character who has gone skydiving exactly once and continues to use that strategy every time he's in a fight... The number of bodies found piled up in that one spot would start to get a little suspicious...
          $endgroup$
          – Darrel Hoffman
          13 hours ago






        • 1




          $begingroup$
          All the person has to do is take a flight across the Atlantic or Pacific. Heck, even a cruise ship will do but it's not as certain. Then they can teleport themselves and the target somewhere where no one will find the body. With the airline flight, the teleporter would be so high that he would have to deal with very low airpressure and extreme cold for a fraction of a second. Off of a cruise ship, the kill isn't certain since they arrive closer to the water and in a shipping lane.
          $endgroup$
          – ShadoCat
          13 hours ago






        • 2




          $begingroup$
          The Earth is rotating and orbiting the Sun, and the Milky Way (with the Earth and the Sun in it) is also rotating. Given the density of space, the location he has been to is now with very high probability in outer space, and humans can't survive the cold and the lack of air for more than a minute. This is practical for taking someone to outer space, and thus quickly killing them, but then teleporting back to a safe place will be a problem.
          $endgroup$
          – pts
          13 hours ago








        • 1




          $begingroup$
          @pts Currently we have no absolute reference frame, that is, no point in space which is absolutely "not moving". (We may find one in the future, but probably not.) So we only have relative reference frames to choose from. At the very least, Teleporters will teleport with reference to the Earth, if not with reference to their destination e.g. a moving vehicle.
          $endgroup$
          – Martin Carney
          12 hours ago
















        $begingroup$
        I'm just considering the implications of a Nightcrawler-like character who has gone skydiving exactly once and continues to use that strategy every time he's in a fight... The number of bodies found piled up in that one spot would start to get a little suspicious...
        $endgroup$
        – Darrel Hoffman
        13 hours ago




        $begingroup$
        I'm just considering the implications of a Nightcrawler-like character who has gone skydiving exactly once and continues to use that strategy every time he's in a fight... The number of bodies found piled up in that one spot would start to get a little suspicious...
        $endgroup$
        – Darrel Hoffman
        13 hours ago




        1




        1




        $begingroup$
        All the person has to do is take a flight across the Atlantic or Pacific. Heck, even a cruise ship will do but it's not as certain. Then they can teleport themselves and the target somewhere where no one will find the body. With the airline flight, the teleporter would be so high that he would have to deal with very low airpressure and extreme cold for a fraction of a second. Off of a cruise ship, the kill isn't certain since they arrive closer to the water and in a shipping lane.
        $endgroup$
        – ShadoCat
        13 hours ago




        $begingroup$
        All the person has to do is take a flight across the Atlantic or Pacific. Heck, even a cruise ship will do but it's not as certain. Then they can teleport themselves and the target somewhere where no one will find the body. With the airline flight, the teleporter would be so high that he would have to deal with very low airpressure and extreme cold for a fraction of a second. Off of a cruise ship, the kill isn't certain since they arrive closer to the water and in a shipping lane.
        $endgroup$
        – ShadoCat
        13 hours ago




        2




        2




        $begingroup$
        The Earth is rotating and orbiting the Sun, and the Milky Way (with the Earth and the Sun in it) is also rotating. Given the density of space, the location he has been to is now with very high probability in outer space, and humans can't survive the cold and the lack of air for more than a minute. This is practical for taking someone to outer space, and thus quickly killing them, but then teleporting back to a safe place will be a problem.
        $endgroup$
        – pts
        13 hours ago






        $begingroup$
        The Earth is rotating and orbiting the Sun, and the Milky Way (with the Earth and the Sun in it) is also rotating. Given the density of space, the location he has been to is now with very high probability in outer space, and humans can't survive the cold and the lack of air for more than a minute. This is practical for taking someone to outer space, and thus quickly killing them, but then teleporting back to a safe place will be a problem.
        $endgroup$
        – pts
        13 hours ago






        1




        1




        $begingroup$
        @pts Currently we have no absolute reference frame, that is, no point in space which is absolutely "not moving". (We may find one in the future, but probably not.) So we only have relative reference frames to choose from. At the very least, Teleporters will teleport with reference to the Earth, if not with reference to their destination e.g. a moving vehicle.
        $endgroup$
        – Martin Carney
        12 hours ago




        $begingroup$
        @pts Currently we have no absolute reference frame, that is, no point in space which is absolutely "not moving". (We may find one in the future, but probably not.) So we only have relative reference frames to choose from. At the very least, Teleporters will teleport with reference to the Earth, if not with reference to their destination e.g. a moving vehicle.
        $endgroup$
        – Martin Carney
        12 hours ago











        4












        $begingroup$

        Unfortunately, the limitation of places he can teleport to by having to have been there at some point makes him a better defensive fighter than an offensive. Having sufficient time to prepare, he can literally attack from anywhere within an area.



        One on One.



        Your teleporter should teleport twice his mass in an entangling object over a wide area (nets, barbed wire, fishing line) to immobilize his opponent. Then, teleport a pointy weighted object on top of them. Having been entangled they will have a hard time dodging and each dodge they succeed will make them more entangled and less able to dodge the next one. Alternatively, teleport behind them and touch them to teleport them to a location that you have sky-dived (without a parachute).



        Multiple opponents.



        Survival should be paramount in a one on many fight. Teleport and touch a combatant and put them literally anywhere other than the fighting location until it is 1v1. You could even use the mid-air approach from previously. The key is to not fight disadvantageous fights and instead make every fight fair or in your favor.



        Mass Destruction.



        No more so than the average person. The only caveat is that what would normally be a suicide attack for most people is now survivable via teleportation.



        Most people would be unable to track and/or stop your teleporter. If he can teleport as fast as he can think then he would be able to (with practice) eliminate people within seconds.






        share|improve this answer









        $endgroup$













        • $begingroup$
          Kamikaze teleporters sounds cool.
          $endgroup$
          – Malkev
          15 hours ago










        • $begingroup$
          Huh, You made me think of something. What if he concentrates hard enough to constantly teleport anything that touches him for the most miniscule of moments, therefore gaining invulnerability from shrapnel and bullets alike because they will not have enough time to impart energy!
          $endgroup$
          – IT Alex
          11 hours ago
















        4












        $begingroup$

        Unfortunately, the limitation of places he can teleport to by having to have been there at some point makes him a better defensive fighter than an offensive. Having sufficient time to prepare, he can literally attack from anywhere within an area.



        One on One.



        Your teleporter should teleport twice his mass in an entangling object over a wide area (nets, barbed wire, fishing line) to immobilize his opponent. Then, teleport a pointy weighted object on top of them. Having been entangled they will have a hard time dodging and each dodge they succeed will make them more entangled and less able to dodge the next one. Alternatively, teleport behind them and touch them to teleport them to a location that you have sky-dived (without a parachute).



        Multiple opponents.



        Survival should be paramount in a one on many fight. Teleport and touch a combatant and put them literally anywhere other than the fighting location until it is 1v1. You could even use the mid-air approach from previously. The key is to not fight disadvantageous fights and instead make every fight fair or in your favor.



        Mass Destruction.



        No more so than the average person. The only caveat is that what would normally be a suicide attack for most people is now survivable via teleportation.



        Most people would be unable to track and/or stop your teleporter. If he can teleport as fast as he can think then he would be able to (with practice) eliminate people within seconds.






        share|improve this answer









        $endgroup$













        • $begingroup$
          Kamikaze teleporters sounds cool.
          $endgroup$
          – Malkev
          15 hours ago










        • $begingroup$
          Huh, You made me think of something. What if he concentrates hard enough to constantly teleport anything that touches him for the most miniscule of moments, therefore gaining invulnerability from shrapnel and bullets alike because they will not have enough time to impart energy!
          $endgroup$
          – IT Alex
          11 hours ago














        4












        4








        4





        $begingroup$

        Unfortunately, the limitation of places he can teleport to by having to have been there at some point makes him a better defensive fighter than an offensive. Having sufficient time to prepare, he can literally attack from anywhere within an area.



        One on One.



        Your teleporter should teleport twice his mass in an entangling object over a wide area (nets, barbed wire, fishing line) to immobilize his opponent. Then, teleport a pointy weighted object on top of them. Having been entangled they will have a hard time dodging and each dodge they succeed will make them more entangled and less able to dodge the next one. Alternatively, teleport behind them and touch them to teleport them to a location that you have sky-dived (without a parachute).



        Multiple opponents.



        Survival should be paramount in a one on many fight. Teleport and touch a combatant and put them literally anywhere other than the fighting location until it is 1v1. You could even use the mid-air approach from previously. The key is to not fight disadvantageous fights and instead make every fight fair or in your favor.



        Mass Destruction.



        No more so than the average person. The only caveat is that what would normally be a suicide attack for most people is now survivable via teleportation.



        Most people would be unable to track and/or stop your teleporter. If he can teleport as fast as he can think then he would be able to (with practice) eliminate people within seconds.






        share|improve this answer









        $endgroup$



        Unfortunately, the limitation of places he can teleport to by having to have been there at some point makes him a better defensive fighter than an offensive. Having sufficient time to prepare, he can literally attack from anywhere within an area.



        One on One.



        Your teleporter should teleport twice his mass in an entangling object over a wide area (nets, barbed wire, fishing line) to immobilize his opponent. Then, teleport a pointy weighted object on top of them. Having been entangled they will have a hard time dodging and each dodge they succeed will make them more entangled and less able to dodge the next one. Alternatively, teleport behind them and touch them to teleport them to a location that you have sky-dived (without a parachute).



        Multiple opponents.



        Survival should be paramount in a one on many fight. Teleport and touch a combatant and put them literally anywhere other than the fighting location until it is 1v1. You could even use the mid-air approach from previously. The key is to not fight disadvantageous fights and instead make every fight fair or in your favor.



        Mass Destruction.



        No more so than the average person. The only caveat is that what would normally be a suicide attack for most people is now survivable via teleportation.



        Most people would be unable to track and/or stop your teleporter. If he can teleport as fast as he can think then he would be able to (with practice) eliminate people within seconds.







        share|improve this answer












        share|improve this answer



        share|improve this answer










        answered 16 hours ago









        IT AlexIT Alex

        8389




        8389












        • $begingroup$
          Kamikaze teleporters sounds cool.
          $endgroup$
          – Malkev
          15 hours ago










        • $begingroup$
          Huh, You made me think of something. What if he concentrates hard enough to constantly teleport anything that touches him for the most miniscule of moments, therefore gaining invulnerability from shrapnel and bullets alike because they will not have enough time to impart energy!
          $endgroup$
          – IT Alex
          11 hours ago


















        • $begingroup$
          Kamikaze teleporters sounds cool.
          $endgroup$
          – Malkev
          15 hours ago










        • $begingroup$
          Huh, You made me think of something. What if he concentrates hard enough to constantly teleport anything that touches him for the most miniscule of moments, therefore gaining invulnerability from shrapnel and bullets alike because they will not have enough time to impart energy!
          $endgroup$
          – IT Alex
          11 hours ago
















        $begingroup$
        Kamikaze teleporters sounds cool.
        $endgroup$
        – Malkev
        15 hours ago




        $begingroup$
        Kamikaze teleporters sounds cool.
        $endgroup$
        – Malkev
        15 hours ago












        $begingroup$
        Huh, You made me think of something. What if he concentrates hard enough to constantly teleport anything that touches him for the most miniscule of moments, therefore gaining invulnerability from shrapnel and bullets alike because they will not have enough time to impart energy!
        $endgroup$
        – IT Alex
        11 hours ago




        $begingroup$
        Huh, You made me think of something. What if he concentrates hard enough to constantly teleport anything that touches him for the most miniscule of moments, therefore gaining invulnerability from shrapnel and bullets alike because they will not have enough time to impart energy!
        $endgroup$
        – IT Alex
        11 hours ago











        4












        $begingroup$

        Well, one less lethal thing no one else has pointed out is instant incarceration. The teleporter visits a jail cell or a bank vault, which is then locked. The teleporter doesn't even need to be let out, of course, because they can take themselves out at any time.



        Once in conflict with someone, the teleporter need only touch their opponent to teleport them away, into the locked cell or vault. (Go to jail. Go directly to jail. Do not pass Go. Do not collect $200.)



        For a more lethal version, the teleporter need only enter an incinerator when it's not on, and then have it fired up after they leave. (Tap. You're toast.) This idea could be extended to any dangerous environment where the danger can be easily turned off and on: inside an nuclear reactor, directly above an industrial metal shredder, a large container filled with acid or toxic gas, etc. Heck, the teleporter could even go in a deep sea submarine once, and then teleport people to the bottom of the ocean.






        share|improve this answer









        $endgroup$













        • $begingroup$
          The pragmatist in me would book a trip on the deepest submersible possible. It would guarantee death (I think the sudden pressure change would kill someone long before drowning) and dispose of the body simultaneously. However, I would rather book a trip into space so I can drop them into LEO. I expect they would fall back to Earth and leave a mess for people to investigate, but it would be way cooler! Besides, who doesn't want to go to space?
          $endgroup$
          – conman
          6 hours ago
















        4












        $begingroup$

        Well, one less lethal thing no one else has pointed out is instant incarceration. The teleporter visits a jail cell or a bank vault, which is then locked. The teleporter doesn't even need to be let out, of course, because they can take themselves out at any time.



        Once in conflict with someone, the teleporter need only touch their opponent to teleport them away, into the locked cell or vault. (Go to jail. Go directly to jail. Do not pass Go. Do not collect $200.)



        For a more lethal version, the teleporter need only enter an incinerator when it's not on, and then have it fired up after they leave. (Tap. You're toast.) This idea could be extended to any dangerous environment where the danger can be easily turned off and on: inside an nuclear reactor, directly above an industrial metal shredder, a large container filled with acid or toxic gas, etc. Heck, the teleporter could even go in a deep sea submarine once, and then teleport people to the bottom of the ocean.






        share|improve this answer









        $endgroup$













        • $begingroup$
          The pragmatist in me would book a trip on the deepest submersible possible. It would guarantee death (I think the sudden pressure change would kill someone long before drowning) and dispose of the body simultaneously. However, I would rather book a trip into space so I can drop them into LEO. I expect they would fall back to Earth and leave a mess for people to investigate, but it would be way cooler! Besides, who doesn't want to go to space?
          $endgroup$
          – conman
          6 hours ago














        4












        4








        4





        $begingroup$

        Well, one less lethal thing no one else has pointed out is instant incarceration. The teleporter visits a jail cell or a bank vault, which is then locked. The teleporter doesn't even need to be let out, of course, because they can take themselves out at any time.



        Once in conflict with someone, the teleporter need only touch their opponent to teleport them away, into the locked cell or vault. (Go to jail. Go directly to jail. Do not pass Go. Do not collect $200.)



        For a more lethal version, the teleporter need only enter an incinerator when it's not on, and then have it fired up after they leave. (Tap. You're toast.) This idea could be extended to any dangerous environment where the danger can be easily turned off and on: inside an nuclear reactor, directly above an industrial metal shredder, a large container filled with acid or toxic gas, etc. Heck, the teleporter could even go in a deep sea submarine once, and then teleport people to the bottom of the ocean.






        share|improve this answer









        $endgroup$



        Well, one less lethal thing no one else has pointed out is instant incarceration. The teleporter visits a jail cell or a bank vault, which is then locked. The teleporter doesn't even need to be let out, of course, because they can take themselves out at any time.



        Once in conflict with someone, the teleporter need only touch their opponent to teleport them away, into the locked cell or vault. (Go to jail. Go directly to jail. Do not pass Go. Do not collect $200.)



        For a more lethal version, the teleporter need only enter an incinerator when it's not on, and then have it fired up after they leave. (Tap. You're toast.) This idea could be extended to any dangerous environment where the danger can be easily turned off and on: inside an nuclear reactor, directly above an industrial metal shredder, a large container filled with acid or toxic gas, etc. Heck, the teleporter could even go in a deep sea submarine once, and then teleport people to the bottom of the ocean.







        share|improve this answer












        share|improve this answer



        share|improve this answer










        answered 6 hours ago









        Earl JenkinsEarl Jenkins

        36114




        36114












        • $begingroup$
          The pragmatist in me would book a trip on the deepest submersible possible. It would guarantee death (I think the sudden pressure change would kill someone long before drowning) and dispose of the body simultaneously. However, I would rather book a trip into space so I can drop them into LEO. I expect they would fall back to Earth and leave a mess for people to investigate, but it would be way cooler! Besides, who doesn't want to go to space?
          $endgroup$
          – conman
          6 hours ago


















        • $begingroup$
          The pragmatist in me would book a trip on the deepest submersible possible. It would guarantee death (I think the sudden pressure change would kill someone long before drowning) and dispose of the body simultaneously. However, I would rather book a trip into space so I can drop them into LEO. I expect they would fall back to Earth and leave a mess for people to investigate, but it would be way cooler! Besides, who doesn't want to go to space?
          $endgroup$
          – conman
          6 hours ago
















        $begingroup$
        The pragmatist in me would book a trip on the deepest submersible possible. It would guarantee death (I think the sudden pressure change would kill someone long before drowning) and dispose of the body simultaneously. However, I would rather book a trip into space so I can drop them into LEO. I expect they would fall back to Earth and leave a mess for people to investigate, but it would be way cooler! Besides, who doesn't want to go to space?
        $endgroup$
        – conman
        6 hours ago




        $begingroup$
        The pragmatist in me would book a trip on the deepest submersible possible. It would guarantee death (I think the sudden pressure change would kill someone long before drowning) and dispose of the body simultaneously. However, I would rather book a trip into space so I can drop them into LEO. I expect they would fall back to Earth and leave a mess for people to investigate, but it would be way cooler! Besides, who doesn't want to go to space?
        $endgroup$
        – conman
        6 hours ago











        3












        $begingroup$

        My answer works on Three assumptions:



        1: The person using the teleportation power can teleport parts of a object he/she is touching.



        And/Or



        2: There is little/no cool-down in-between teleportation attempts.



        And/Or



        3: If the teleporter can choose how precise the "location he has been" part is.



        Most organisms cannot function in combat without a brain. So if assumption #1 is correct, then you can not only quickly kill them, but also merely incapacitate them by removing things such as clothing, weapons, limbs, or possibly just all the air in their lungs.



        All people need a defense, whether it is armor, wits, luck, or a combination of said three. Teleporters have a fourth if assumption #2 works: By constantly teleporting the area near them away, you create what essentially is a wormhole that they control in order to not only prevent damage, but maybe also send the damage right back at the aggressors!



        If you have in combat and holding a sword (example item), you could teleport the hilt of the sword right next to the opponent's forehead, with the blade orientated towards the brain. Voila! Sword has now been stabbed though whatever armor the guy has, and is now in the brain.



        This answer focuses on on-on-one and can be extended to one-on-multiple.






        share|improve this answer









        $endgroup$


















          3












          $begingroup$

          My answer works on Three assumptions:



          1: The person using the teleportation power can teleport parts of a object he/she is touching.



          And/Or



          2: There is little/no cool-down in-between teleportation attempts.



          And/Or



          3: If the teleporter can choose how precise the "location he has been" part is.



          Most organisms cannot function in combat without a brain. So if assumption #1 is correct, then you can not only quickly kill them, but also merely incapacitate them by removing things such as clothing, weapons, limbs, or possibly just all the air in their lungs.



          All people need a defense, whether it is armor, wits, luck, or a combination of said three. Teleporters have a fourth if assumption #2 works: By constantly teleporting the area near them away, you create what essentially is a wormhole that they control in order to not only prevent damage, but maybe also send the damage right back at the aggressors!



          If you have in combat and holding a sword (example item), you could teleport the hilt of the sword right next to the opponent's forehead, with the blade orientated towards the brain. Voila! Sword has now been stabbed though whatever armor the guy has, and is now in the brain.



          This answer focuses on on-on-one and can be extended to one-on-multiple.






          share|improve this answer









          $endgroup$
















            3












            3








            3





            $begingroup$

            My answer works on Three assumptions:



            1: The person using the teleportation power can teleport parts of a object he/she is touching.



            And/Or



            2: There is little/no cool-down in-between teleportation attempts.



            And/Or



            3: If the teleporter can choose how precise the "location he has been" part is.



            Most organisms cannot function in combat without a brain. So if assumption #1 is correct, then you can not only quickly kill them, but also merely incapacitate them by removing things such as clothing, weapons, limbs, or possibly just all the air in their lungs.



            All people need a defense, whether it is armor, wits, luck, or a combination of said three. Teleporters have a fourth if assumption #2 works: By constantly teleporting the area near them away, you create what essentially is a wormhole that they control in order to not only prevent damage, but maybe also send the damage right back at the aggressors!



            If you have in combat and holding a sword (example item), you could teleport the hilt of the sword right next to the opponent's forehead, with the blade orientated towards the brain. Voila! Sword has now been stabbed though whatever armor the guy has, and is now in the brain.



            This answer focuses on on-on-one and can be extended to one-on-multiple.






            share|improve this answer









            $endgroup$



            My answer works on Three assumptions:



            1: The person using the teleportation power can teleport parts of a object he/she is touching.



            And/Or



            2: There is little/no cool-down in-between teleportation attempts.



            And/Or



            3: If the teleporter can choose how precise the "location he has been" part is.



            Most organisms cannot function in combat without a brain. So if assumption #1 is correct, then you can not only quickly kill them, but also merely incapacitate them by removing things such as clothing, weapons, limbs, or possibly just all the air in their lungs.



            All people need a defense, whether it is armor, wits, luck, or a combination of said three. Teleporters have a fourth if assumption #2 works: By constantly teleporting the area near them away, you create what essentially is a wormhole that they control in order to not only prevent damage, but maybe also send the damage right back at the aggressors!



            If you have in combat and holding a sword (example item), you could teleport the hilt of the sword right next to the opponent's forehead, with the blade orientated towards the brain. Voila! Sword has now been stabbed though whatever armor the guy has, and is now in the brain.



            This answer focuses on on-on-one and can be extended to one-on-multiple.







            share|improve this answer












            share|improve this answer



            share|improve this answer










            answered 14 hours ago









            Joe PJoe P

            836515




            836515























                3












                $begingroup$

                This power would be extraordinarily useful. Anywhere this person has ever been, they can make "anything they touch" appear there.



                Convenient death trap: arrange to send folks to kill zones



                Make an expedition to the North Pole (which will kill probably even many supers unprepared to be sent there in very short order). Take an airplane ride with a window seat, then simply teleport anyone the person wants to get rid of there.



                Or just get a secret lair. Build a sub-sub basement. Surround it with chain link fencing, then flood it with water. Teleport people there to drown. Much less public that way. People just disappear and aren't found in spectacular, publicity-generating ways (like falling out of the sky randomly over the Midwest, or a huge pile of people killed via exposure being found at the North Pole).



                If the user doesn't want to kill their enemies, and has resources, like they're Batman, build a prison. Hire a bunch of superpowered guards. Spend a few minutes locked inside a 2-foot by 2-foot square closet surrounded by powerproof glass, and you've got an intake cell where your new inmate materializes. Repeat, and you can handle multiple incoming bad (or I suppose good) guys simultaneously.



                Anywhere the user has, or might have ever, been is not secure



                Anywhere they can break into, or might have ever broken into, is not secure. Any place they might have flim-flammed their way past the front desk in disguise (think civvies), even briefly, is not secure. Any place they've ever worked cannot be considered secure. Note that the list of such places is vast because you won't know where they've been. This remains true as long as they live.



                They can take a tour of the White House, even the unsecured parts, in disguise. Twenty years later, they could decide 'screw this, time to assassinate the President' and now you've got a problem. They can take a job as the janitor of an office building where someone they're targeting works. Failing everything else, they take a job working for these guys and then pretty much learn how to get anywhere they want.



                Seriously, watch the video, it exposes how much even 'secure' locks mostly aren't. All this person needs to do is pop a door when nobody's around, or maybe when people are around and you just do the 'elevator repairman' schtick (again from the video).



                TL;DR - This person can get almost anywhere. And if they can, then anyone who can engage their services can also. If you want in, and either can pay, or convince them to help you, a plan can almost certainly be arranged to give the teleport user the ability to get your entire crew in. This person can also assassinate a lot of folks by finding their office, then teleporting hand grenades or nerve gas canisters or even assassin droids in.






                share|improve this answer









                $endgroup$













                • $begingroup$
                  Have their 'secret identity' - or one of their secret identities - explicitly be as a pen tester. Then you can essentially hand wave their ability to get somewhere that other folks would really rather they didn't.
                  $endgroup$
                  – Dayton Williams
                  5 hours ago
















                3












                $begingroup$

                This power would be extraordinarily useful. Anywhere this person has ever been, they can make "anything they touch" appear there.



                Convenient death trap: arrange to send folks to kill zones



                Make an expedition to the North Pole (which will kill probably even many supers unprepared to be sent there in very short order). Take an airplane ride with a window seat, then simply teleport anyone the person wants to get rid of there.



                Or just get a secret lair. Build a sub-sub basement. Surround it with chain link fencing, then flood it with water. Teleport people there to drown. Much less public that way. People just disappear and aren't found in spectacular, publicity-generating ways (like falling out of the sky randomly over the Midwest, or a huge pile of people killed via exposure being found at the North Pole).



                If the user doesn't want to kill their enemies, and has resources, like they're Batman, build a prison. Hire a bunch of superpowered guards. Spend a few minutes locked inside a 2-foot by 2-foot square closet surrounded by powerproof glass, and you've got an intake cell where your new inmate materializes. Repeat, and you can handle multiple incoming bad (or I suppose good) guys simultaneously.



                Anywhere the user has, or might have ever, been is not secure



                Anywhere they can break into, or might have ever broken into, is not secure. Any place they might have flim-flammed their way past the front desk in disguise (think civvies), even briefly, is not secure. Any place they've ever worked cannot be considered secure. Note that the list of such places is vast because you won't know where they've been. This remains true as long as they live.



                They can take a tour of the White House, even the unsecured parts, in disguise. Twenty years later, they could decide 'screw this, time to assassinate the President' and now you've got a problem. They can take a job as the janitor of an office building where someone they're targeting works. Failing everything else, they take a job working for these guys and then pretty much learn how to get anywhere they want.



                Seriously, watch the video, it exposes how much even 'secure' locks mostly aren't. All this person needs to do is pop a door when nobody's around, or maybe when people are around and you just do the 'elevator repairman' schtick (again from the video).



                TL;DR - This person can get almost anywhere. And if they can, then anyone who can engage their services can also. If you want in, and either can pay, or convince them to help you, a plan can almost certainly be arranged to give the teleport user the ability to get your entire crew in. This person can also assassinate a lot of folks by finding their office, then teleporting hand grenades or nerve gas canisters or even assassin droids in.






                share|improve this answer









                $endgroup$













                • $begingroup$
                  Have their 'secret identity' - or one of their secret identities - explicitly be as a pen tester. Then you can essentially hand wave their ability to get somewhere that other folks would really rather they didn't.
                  $endgroup$
                  – Dayton Williams
                  5 hours ago














                3












                3








                3





                $begingroup$

                This power would be extraordinarily useful. Anywhere this person has ever been, they can make "anything they touch" appear there.



                Convenient death trap: arrange to send folks to kill zones



                Make an expedition to the North Pole (which will kill probably even many supers unprepared to be sent there in very short order). Take an airplane ride with a window seat, then simply teleport anyone the person wants to get rid of there.



                Or just get a secret lair. Build a sub-sub basement. Surround it with chain link fencing, then flood it with water. Teleport people there to drown. Much less public that way. People just disappear and aren't found in spectacular, publicity-generating ways (like falling out of the sky randomly over the Midwest, or a huge pile of people killed via exposure being found at the North Pole).



                If the user doesn't want to kill their enemies, and has resources, like they're Batman, build a prison. Hire a bunch of superpowered guards. Spend a few minutes locked inside a 2-foot by 2-foot square closet surrounded by powerproof glass, and you've got an intake cell where your new inmate materializes. Repeat, and you can handle multiple incoming bad (or I suppose good) guys simultaneously.



                Anywhere the user has, or might have ever, been is not secure



                Anywhere they can break into, or might have ever broken into, is not secure. Any place they might have flim-flammed their way past the front desk in disguise (think civvies), even briefly, is not secure. Any place they've ever worked cannot be considered secure. Note that the list of such places is vast because you won't know where they've been. This remains true as long as they live.



                They can take a tour of the White House, even the unsecured parts, in disguise. Twenty years later, they could decide 'screw this, time to assassinate the President' and now you've got a problem. They can take a job as the janitor of an office building where someone they're targeting works. Failing everything else, they take a job working for these guys and then pretty much learn how to get anywhere they want.



                Seriously, watch the video, it exposes how much even 'secure' locks mostly aren't. All this person needs to do is pop a door when nobody's around, or maybe when people are around and you just do the 'elevator repairman' schtick (again from the video).



                TL;DR - This person can get almost anywhere. And if they can, then anyone who can engage their services can also. If you want in, and either can pay, or convince them to help you, a plan can almost certainly be arranged to give the teleport user the ability to get your entire crew in. This person can also assassinate a lot of folks by finding their office, then teleporting hand grenades or nerve gas canisters or even assassin droids in.






                share|improve this answer









                $endgroup$



                This power would be extraordinarily useful. Anywhere this person has ever been, they can make "anything they touch" appear there.



                Convenient death trap: arrange to send folks to kill zones



                Make an expedition to the North Pole (which will kill probably even many supers unprepared to be sent there in very short order). Take an airplane ride with a window seat, then simply teleport anyone the person wants to get rid of there.



                Or just get a secret lair. Build a sub-sub basement. Surround it with chain link fencing, then flood it with water. Teleport people there to drown. Much less public that way. People just disappear and aren't found in spectacular, publicity-generating ways (like falling out of the sky randomly over the Midwest, or a huge pile of people killed via exposure being found at the North Pole).



                If the user doesn't want to kill their enemies, and has resources, like they're Batman, build a prison. Hire a bunch of superpowered guards. Spend a few minutes locked inside a 2-foot by 2-foot square closet surrounded by powerproof glass, and you've got an intake cell where your new inmate materializes. Repeat, and you can handle multiple incoming bad (or I suppose good) guys simultaneously.



                Anywhere the user has, or might have ever, been is not secure



                Anywhere they can break into, or might have ever broken into, is not secure. Any place they might have flim-flammed their way past the front desk in disguise (think civvies), even briefly, is not secure. Any place they've ever worked cannot be considered secure. Note that the list of such places is vast because you won't know where they've been. This remains true as long as they live.



                They can take a tour of the White House, even the unsecured parts, in disguise. Twenty years later, they could decide 'screw this, time to assassinate the President' and now you've got a problem. They can take a job as the janitor of an office building where someone they're targeting works. Failing everything else, they take a job working for these guys and then pretty much learn how to get anywhere they want.



                Seriously, watch the video, it exposes how much even 'secure' locks mostly aren't. All this person needs to do is pop a door when nobody's around, or maybe when people are around and you just do the 'elevator repairman' schtick (again from the video).



                TL;DR - This person can get almost anywhere. And if they can, then anyone who can engage their services can also. If you want in, and either can pay, or convince them to help you, a plan can almost certainly be arranged to give the teleport user the ability to get your entire crew in. This person can also assassinate a lot of folks by finding their office, then teleporting hand grenades or nerve gas canisters or even assassin droids in.







                share|improve this answer












                share|improve this answer



                share|improve this answer










                answered 5 hours ago









                Dayton WilliamsDayton Williams

                1,921316




                1,921316












                • $begingroup$
                  Have their 'secret identity' - or one of their secret identities - explicitly be as a pen tester. Then you can essentially hand wave their ability to get somewhere that other folks would really rather they didn't.
                  $endgroup$
                  – Dayton Williams
                  5 hours ago


















                • $begingroup$
                  Have their 'secret identity' - or one of their secret identities - explicitly be as a pen tester. Then you can essentially hand wave their ability to get somewhere that other folks would really rather they didn't.
                  $endgroup$
                  – Dayton Williams
                  5 hours ago
















                $begingroup$
                Have their 'secret identity' - or one of their secret identities - explicitly be as a pen tester. Then you can essentially hand wave their ability to get somewhere that other folks would really rather they didn't.
                $endgroup$
                – Dayton Williams
                5 hours ago




                $begingroup$
                Have their 'secret identity' - or one of their secret identities - explicitly be as a pen tester. Then you can essentially hand wave their ability to get somewhere that other folks would really rather they didn't.
                $endgroup$
                – Dayton Williams
                5 hours ago











                2












                $begingroup$

                Other answers have covered fairly well the options for one to one combat, and even vs groups. This answer poses a solution to mass destruction.



                The only restriction is enabling partial teleportation.



                Here's a few ideas, depending on how extreme you want to go.



                1) Shockwaves. Teleportation should always produce a shockwave. Consider, what happens to the air that used to occupy the space the teleported object now does? As you've defined the ability (must be touching thing to teleport) they could not possibly swap the air and the teleported object. So the air must remain, pushed out at hyper velocity by the sudden presence of something teleporting in. Boom, shockwave!



                2) Since you are already moving matter out of the way (air) what happens if you teleport an object into something more... substantial? Imagine teleporting a 200 pound block directly into the same space already occupied by another 200 pound block. A rather impressive explosion should result as most of the chemical bonds in whatever used to be there are suddenly broken all at once. At the very least, it generates a shockwave as before, only with a large amount of shrapnel mixed in.



                3) Let's take his to the logical extreme: use simulations partial teleportations to compress an object down to a single point. Literally. Depending on how precise and accurate your teleporter can be, this should result in a nuclear explosion, as the an entire 200 pounds of atomic nuclei are forced into the same space, destabilizing them and causing them to break up. This should result in a nuclear explosion. Depending on material teleported, it may just result in irradiating the surrounding area with exotic particles (you might even want this outcome, kill the people, leave the buildings...). Alternatively, it also might form a black hole and destroy the Earth. Be careful with this one...



                As a disclaimer, I haven't run the numbers on any of these situations. They just seem like logical consequences based on my understanding of physics. I'm be happy to be corrected if someone else cares to do so.






                share|improve this answer









                $endgroup$













                • $begingroup$
                  The only downside I see is that you have to be touching the teleported stuff. Presumably at both ends.
                  $endgroup$
                  – IT Alex
                  12 hours ago










                • $begingroup$
                  Ah, yes, that would make a difference. I was assuming you had to be touching it on the sending end, not the receiving end. Still, if there's no cool down, the teleporter might have time to jump away in time...?
                  $endgroup$
                  – jpgo5000
                  11 hours ago
















                2












                $begingroup$

                Other answers have covered fairly well the options for one to one combat, and even vs groups. This answer poses a solution to mass destruction.



                The only restriction is enabling partial teleportation.



                Here's a few ideas, depending on how extreme you want to go.



                1) Shockwaves. Teleportation should always produce a shockwave. Consider, what happens to the air that used to occupy the space the teleported object now does? As you've defined the ability (must be touching thing to teleport) they could not possibly swap the air and the teleported object. So the air must remain, pushed out at hyper velocity by the sudden presence of something teleporting in. Boom, shockwave!



                2) Since you are already moving matter out of the way (air) what happens if you teleport an object into something more... substantial? Imagine teleporting a 200 pound block directly into the same space already occupied by another 200 pound block. A rather impressive explosion should result as most of the chemical bonds in whatever used to be there are suddenly broken all at once. At the very least, it generates a shockwave as before, only with a large amount of shrapnel mixed in.



                3) Let's take his to the logical extreme: use simulations partial teleportations to compress an object down to a single point. Literally. Depending on how precise and accurate your teleporter can be, this should result in a nuclear explosion, as the an entire 200 pounds of atomic nuclei are forced into the same space, destabilizing them and causing them to break up. This should result in a nuclear explosion. Depending on material teleported, it may just result in irradiating the surrounding area with exotic particles (you might even want this outcome, kill the people, leave the buildings...). Alternatively, it also might form a black hole and destroy the Earth. Be careful with this one...



                As a disclaimer, I haven't run the numbers on any of these situations. They just seem like logical consequences based on my understanding of physics. I'm be happy to be corrected if someone else cares to do so.






                share|improve this answer









                $endgroup$













                • $begingroup$
                  The only downside I see is that you have to be touching the teleported stuff. Presumably at both ends.
                  $endgroup$
                  – IT Alex
                  12 hours ago










                • $begingroup$
                  Ah, yes, that would make a difference. I was assuming you had to be touching it on the sending end, not the receiving end. Still, if there's no cool down, the teleporter might have time to jump away in time...?
                  $endgroup$
                  – jpgo5000
                  11 hours ago














                2












                2








                2





                $begingroup$

                Other answers have covered fairly well the options for one to one combat, and even vs groups. This answer poses a solution to mass destruction.



                The only restriction is enabling partial teleportation.



                Here's a few ideas, depending on how extreme you want to go.



                1) Shockwaves. Teleportation should always produce a shockwave. Consider, what happens to the air that used to occupy the space the teleported object now does? As you've defined the ability (must be touching thing to teleport) they could not possibly swap the air and the teleported object. So the air must remain, pushed out at hyper velocity by the sudden presence of something teleporting in. Boom, shockwave!



                2) Since you are already moving matter out of the way (air) what happens if you teleport an object into something more... substantial? Imagine teleporting a 200 pound block directly into the same space already occupied by another 200 pound block. A rather impressive explosion should result as most of the chemical bonds in whatever used to be there are suddenly broken all at once. At the very least, it generates a shockwave as before, only with a large amount of shrapnel mixed in.



                3) Let's take his to the logical extreme: use simulations partial teleportations to compress an object down to a single point. Literally. Depending on how precise and accurate your teleporter can be, this should result in a nuclear explosion, as the an entire 200 pounds of atomic nuclei are forced into the same space, destabilizing them and causing them to break up. This should result in a nuclear explosion. Depending on material teleported, it may just result in irradiating the surrounding area with exotic particles (you might even want this outcome, kill the people, leave the buildings...). Alternatively, it also might form a black hole and destroy the Earth. Be careful with this one...



                As a disclaimer, I haven't run the numbers on any of these situations. They just seem like logical consequences based on my understanding of physics. I'm be happy to be corrected if someone else cares to do so.






                share|improve this answer









                $endgroup$



                Other answers have covered fairly well the options for one to one combat, and even vs groups. This answer poses a solution to mass destruction.



                The only restriction is enabling partial teleportation.



                Here's a few ideas, depending on how extreme you want to go.



                1) Shockwaves. Teleportation should always produce a shockwave. Consider, what happens to the air that used to occupy the space the teleported object now does? As you've defined the ability (must be touching thing to teleport) they could not possibly swap the air and the teleported object. So the air must remain, pushed out at hyper velocity by the sudden presence of something teleporting in. Boom, shockwave!



                2) Since you are already moving matter out of the way (air) what happens if you teleport an object into something more... substantial? Imagine teleporting a 200 pound block directly into the same space already occupied by another 200 pound block. A rather impressive explosion should result as most of the chemical bonds in whatever used to be there are suddenly broken all at once. At the very least, it generates a shockwave as before, only with a large amount of shrapnel mixed in.



                3) Let's take his to the logical extreme: use simulations partial teleportations to compress an object down to a single point. Literally. Depending on how precise and accurate your teleporter can be, this should result in a nuclear explosion, as the an entire 200 pounds of atomic nuclei are forced into the same space, destabilizing them and causing them to break up. This should result in a nuclear explosion. Depending on material teleported, it may just result in irradiating the surrounding area with exotic particles (you might even want this outcome, kill the people, leave the buildings...). Alternatively, it also might form a black hole and destroy the Earth. Be careful with this one...



                As a disclaimer, I haven't run the numbers on any of these situations. They just seem like logical consequences based on my understanding of physics. I'm be happy to be corrected if someone else cares to do so.







                share|improve this answer












                share|improve this answer



                share|improve this answer










                answered 12 hours ago









                jpgo5000jpgo5000

                1615




                1615












                • $begingroup$
                  The only downside I see is that you have to be touching the teleported stuff. Presumably at both ends.
                  $endgroup$
                  – IT Alex
                  12 hours ago










                • $begingroup$
                  Ah, yes, that would make a difference. I was assuming you had to be touching it on the sending end, not the receiving end. Still, if there's no cool down, the teleporter might have time to jump away in time...?
                  $endgroup$
                  – jpgo5000
                  11 hours ago


















                • $begingroup$
                  The only downside I see is that you have to be touching the teleported stuff. Presumably at both ends.
                  $endgroup$
                  – IT Alex
                  12 hours ago










                • $begingroup$
                  Ah, yes, that would make a difference. I was assuming you had to be touching it on the sending end, not the receiving end. Still, if there's no cool down, the teleporter might have time to jump away in time...?
                  $endgroup$
                  – jpgo5000
                  11 hours ago
















                $begingroup$
                The only downside I see is that you have to be touching the teleported stuff. Presumably at both ends.
                $endgroup$
                – IT Alex
                12 hours ago




                $begingroup$
                The only downside I see is that you have to be touching the teleported stuff. Presumably at both ends.
                $endgroup$
                – IT Alex
                12 hours ago












                $begingroup$
                Ah, yes, that would make a difference. I was assuming you had to be touching it on the sending end, not the receiving end. Still, if there's no cool down, the teleporter might have time to jump away in time...?
                $endgroup$
                – jpgo5000
                11 hours ago




                $begingroup$
                Ah, yes, that would make a difference. I was assuming you had to be touching it on the sending end, not the receiving end. Still, if there's no cool down, the teleporter might have time to jump away in time...?
                $endgroup$
                – jpgo5000
                11 hours ago











                0












                $begingroup$

                Not an answer, but you want to check this story out. It's a Harry Potter fanfiction where Harry realizes that Apparation (teleportation for the HP world) is essentially the most important combat tool available in his world. He goes on to focus his prodigious skills into learning extremely fast, accurate, silent, and seemingly-randomized patterns of teleportation which acts as a massive force-multiplier in his battles.



                Definitely worth a read.



                https://www.fanfiction.net/s/7093738/1/Brutal-Harry






                share|improve this answer









                $endgroup$


















                  0












                  $begingroup$

                  Not an answer, but you want to check this story out. It's a Harry Potter fanfiction where Harry realizes that Apparation (teleportation for the HP world) is essentially the most important combat tool available in his world. He goes on to focus his prodigious skills into learning extremely fast, accurate, silent, and seemingly-randomized patterns of teleportation which acts as a massive force-multiplier in his battles.



                  Definitely worth a read.



                  https://www.fanfiction.net/s/7093738/1/Brutal-Harry






                  share|improve this answer









                  $endgroup$
















                    0












                    0








                    0





                    $begingroup$

                    Not an answer, but you want to check this story out. It's a Harry Potter fanfiction where Harry realizes that Apparation (teleportation for the HP world) is essentially the most important combat tool available in his world. He goes on to focus his prodigious skills into learning extremely fast, accurate, silent, and seemingly-randomized patterns of teleportation which acts as a massive force-multiplier in his battles.



                    Definitely worth a read.



                    https://www.fanfiction.net/s/7093738/1/Brutal-Harry






                    share|improve this answer









                    $endgroup$



                    Not an answer, but you want to check this story out. It's a Harry Potter fanfiction where Harry realizes that Apparation (teleportation for the HP world) is essentially the most important combat tool available in his world. He goes on to focus his prodigious skills into learning extremely fast, accurate, silent, and seemingly-randomized patterns of teleportation which acts as a massive force-multiplier in his battles.



                    Definitely worth a read.



                    https://www.fanfiction.net/s/7093738/1/Brutal-Harry







                    share|improve this answer












                    share|improve this answer



                    share|improve this answer










                    answered 9 hours ago









                    MBakMBak

                    213




                    213























                        0












                        $begingroup$

                        I don't know, that sounds overpowered and a bit silly. Teleportation with limits is much more interesting. Like not being able to teleport others out of danger, or not being able to teleport sufficiently far away or more than a few times in quick succession. In Project Horizons there was a character who would teleport behind someone, stab them with a sword, grab their gun and shoot their buddy, use them as a shield, teleport above another guy, crash stomp them, shoot another guy in the head, drop a grenade and teleport out the door and stroll away all while singing a happy song.






                        share|improve this answer









                        $endgroup$


















                          0












                          $begingroup$

                          I don't know, that sounds overpowered and a bit silly. Teleportation with limits is much more interesting. Like not being able to teleport others out of danger, or not being able to teleport sufficiently far away or more than a few times in quick succession. In Project Horizons there was a character who would teleport behind someone, stab them with a sword, grab their gun and shoot their buddy, use them as a shield, teleport above another guy, crash stomp them, shoot another guy in the head, drop a grenade and teleport out the door and stroll away all while singing a happy song.






                          share|improve this answer









                          $endgroup$
















                            0












                            0








                            0





                            $begingroup$

                            I don't know, that sounds overpowered and a bit silly. Teleportation with limits is much more interesting. Like not being able to teleport others out of danger, or not being able to teleport sufficiently far away or more than a few times in quick succession. In Project Horizons there was a character who would teleport behind someone, stab them with a sword, grab their gun and shoot their buddy, use them as a shield, teleport above another guy, crash stomp them, shoot another guy in the head, drop a grenade and teleport out the door and stroll away all while singing a happy song.






                            share|improve this answer









                            $endgroup$



                            I don't know, that sounds overpowered and a bit silly. Teleportation with limits is much more interesting. Like not being able to teleport others out of danger, or not being able to teleport sufficiently far away or more than a few times in quick succession. In Project Horizons there was a character who would teleport behind someone, stab them with a sword, grab their gun and shoot their buddy, use them as a shield, teleport above another guy, crash stomp them, shoot another guy in the head, drop a grenade and teleport out the door and stroll away all while singing a happy song.







                            share|improve this answer












                            share|improve this answer



                            share|improve this answer










                            answered 8 hours ago









                            ggb667ggb667

                            1011




                            1011






























                                draft saved

                                draft discarded




















































                                Thanks for contributing an answer to Worldbuilding Stack Exchange!


                                • Please be sure to answer the question. Provide details and share your research!

                                But avoid



                                • Asking for help, clarification, or responding to other answers.

                                • Making statements based on opinion; back them up with references or personal experience.


                                Use MathJax to format equations. MathJax reference.


                                To learn more, see our tips on writing great answers.




                                draft saved


                                draft discarded














                                StackExchange.ready(
                                function () {
                                StackExchange.openid.initPostLogin('.new-post-login', 'https%3a%2f%2fworldbuilding.stackexchange.com%2fquestions%2f142436%2fpersonal-teleportation-as-a-weapon%23new-answer', 'question_page');
                                }
                                );

                                Post as a guest















                                Required, but never shown





















































                                Required, but never shown














                                Required, but never shown












                                Required, but never shown







                                Required, but never shown

































                                Required, but never shown














                                Required, but never shown












                                Required, but never shown







                                Required, but never shown







                                Popular posts from this blog

                                He _____ here since 1970 . Answer needed [closed]What does “since he was so high” mean?Meaning of “catch birds for”?How do I ensure “since” takes the meaning I want?“Who cares here” meaningWhat does “right round toward” mean?the time tense (had now been detected)What does the phrase “ring around the roses” mean here?Correct usage of “visited upon”Meaning of “foiled rail sabotage bid”It was the third time I had gone to Rome or It is the third time I had been to Rome

                                Bunad

                                Færeyskur hestur Heimild | Tengill | Tilvísanir | LeiðsagnarvalRossið - síða um færeyska hrossið á færeyskuGott ár hjá færeyska hestinum